Download BANK OF TEST TASK Stomatological faculty to the test control for

Document related concepts
no text concepts found
Transcript
BANK OF TEST TASK
to the test control for the year
Stomatological faculty
CYTOLOGY
During embryogenesis the epithelial band also known as vestibular plate gives rise to development of vestibule of mouth. What
biological mechanism of the programmed death of cells provides growth of buccolabial sulcus from epithelial plate?
A *Apoptosis
B Necrosis
C Meiosis
D Paranecrosis
E Amitosis
A patient was prescribed a drug with apparent lipophilic properties. What is the main mechanism of its absorption?
A *Passive diffusion
B Filtration
C Active transport
D Pinocytosis
E Binding with transport proteins
Examination of a patient with hepatolenticular degeneration revealed that synthesis of ceruloplasmin protein has a defect. What
organelles is this defect connected with?
A *Granular endoplasmic reticulum
B Agranular endoplasmic reticulum
C Mitochondrions
D Golgi complex
E Lysosomes
Microscopic analysis of human heart cells revealed some oval oganellas, their tunic being formed by two membranes: the
external one is smooth, and the internal one forms crista. Biochemical analysis determined the presence of ATP-synthetase
enzyme. What organellas were analysed?
A *Mitochondrions
B Lysosomes
C Ribosomes
D Endoplasmic reticulum
E Centrosomes
In process of the secretory cycle secretion granules come and go in the apical part of cytoplasm of pancreas cells. These
granules relate to the following structure elements:
A*Inclusions
B Microfilaments
C Lysosomes
D Exocytic vacuoles
E Granular endoplasmic reticulum
An animal had been intensively fed with carbohydrates. Histologic examination of its liver revealed a significant number of
glycogen granules. Glycogen relates to the following group of cell structures:
A *Trophic granules
B Secretory granules
C Excretory granules
D Pigment granules
E Special organelles
Electron microscopic study of a cell revealed roundish bubbles confined by a membrane and containing a lot of various
hydrolytic enzymes. It is known that these organelles provide intracellular digestion and protective functions. These elements
are:
A *Lysosomes
B Centrosomes
C Endoplasmic reticulum
D Ribosomes
E Mitochondria
Formation of ribosome subunits in a cell was disturbed in course of an experiment (by means of activated mutagenic factors).
This will have an effect on the following metabolic process:
A *Protein biosynthesis
B Carbohydrate biosynthesis
C ATP synthesis
D Photosynthesis
E Biological oxidation
A 50 year old woman had her tooth extracted. The tissue regenerated. Which of the following organelle are the most active
during tissue regeneration?
A *Ribosomes
B Centrioles
C Polysomes
D Smooth endoplasmic reticulum
E Lysosomes
In histological preparation of nervous tissue you can see, that neurons unite are connected together by means of contacts that
are specialized for the one-sided transmission of nervous impulse. Specify, what type of intercellular connection is educed on
preparation?
A *Synapse.
B Desmosome
C Simple
D Tight junctions
E Gap junction
Harmful ecological factors resulted in the sharp falling of endocytosis and exocytosis in the cells of liver and blood. What layer
of cytolemma was suffered first of all?
A *Cortical
B Lipoprotein
C Supra membrane
D Integral
E Glycocalyx
Existence of life at all his levels is determined by the structure of lower level. What level of organization is preceded and
provided the existence of life at cellular level:
A *Molecular
B Tissue
C Organism
D Population
E Biocoenosis
Support of life at any level is related to the phenomenon of reproduction. In what level of organization is reproduction done on
the basis of matrix synthesis?
A * Molecular
B Subcellular
C Cellular
D Tissue
E Levels of organism
During the histological research was found a great number of granules of glycogen in the cells of liver according to the
excessive feeding of animals by carbohydrates. What activity of these organelles indicates this fact?
A *Trophic inclusion
B Secretory inclusion
C Excretory inclusion
D Pigment inclusion
E Non membrane organelles
Two different organelles that destroy the proteins are visible on the electronic photomicrograph of the cell? What kind of
organelles are they?
A *Lysosome and proteasome
B Endoplasmic reticulum and microfilaments
C Peroxisome and ribosome
D Ribosome
E Golgi apparatus and microtubules
During the cytochemical research a light content of hydrolytic enzymes was found in the cytoplasm of cell. What activity of
organelles indicates this fact?
A *Lysosomes
B Endoplasmic reticulum
C Mitochondrion
D Polysome
E Centosome
During the electronic microscopy in the cytoplasm of cell, near the cell nucleus, was found a membranous organelle that
consists of 5-10 flat cisterns, with the extended peripheral areas from which little blisters – (lysosomes) are disconnected.
Name this organelle:
A * Golgi apparatus
B Ribosome
C Mitochondrion
D Cytoskeleton
E Centrosome
On the electronic photo organelle is presented which is a large polyproteasome complex that consists of tube like and two
regulator parts, was located on both ends of organelle. The last performs the function of proteolysis. Name this organelle.
A *Proteasome
B Centriole
C of Inclusions
D Ribosome
E Golgi apparatus
During the studies of epithelial cells of oral cavity on the surface of cell’s nucleus appear the rounded little bodies that
indicated that cells are taken from the oral cavity of woman. How is such a formation of chromatin called?
A *Barr corpuscles
B Herring corpuscles
C Euchromatin
D Pacinian corpuscles
During the forensic research of blood sample in neutrophils on the surface of one of segments of cell’s nucleus the chromatin
appears as a drumstick. How is such structural formation called?
A *Barr corpuscles
B Layon corpuscles
C Chromatin
D Euchromatin
E Pacinian corpuscles
In the certain cells of adults during their lifelong there is no mitosis and quantitative content of DNA remains constant. These
cells are:
A * Neurons
B Endothelia cells
C Muscle (smooth)
D Epidermis
E Blood forming
Analysis of an electron diffraction pattern of a cell revealed mitochondrion destruction. This might result in abnormal course of
the following cell process:
A *Oxidation of organic substances
B Nuclear division
C Crossingover
D Cleavage
E It is known that information about sequence of amino acids in a protein molecule is encoded as a sequence of four types of
nucleotides in a DNA molecule, and different amino acids are encoded by different number of triplets - from one to six. Such
peculiarity of the genetic code is called:
A *Degeneracy
B Universality
C Nonoverlapping
D Triplety
E Specificity
A chemical factor had an effect on the cell membrane. As a result a cell has changed its shape. What layer of cell membrane
was participated in it?
A * Cortical
B Glycocalyx
C Two lipids layer
D Hydrophilic
E Hydrophobic
Sometimes it is necessary to do research of somatic cells for determination of sex of human. What their structures can provide
the information about sex of human?
A *Peripheral chromatin
B Euchromatin
C Decondensation chromatin
D Barr corpuscles
E Optional chromatin
In a muscular tissue there is an intensive aerobic process of accumulation of energy as macro energetic connections ATP. This
process takes place due to the participation of such organelles as:
A *Mitochondrion
B Smooth endoplasmic reticulum
C Lysosomes
D Rough Endoplasmic Reticulum
E Centrosome
Submicroscopic not membranous organelles of general-purpose – microtubules are built from the following protein like:
A *Tubulin
B Desmine
C Keratin
D Vimentine
E Actinin
Centrioles in their basis contained microtubules which are oriented parallel and have such a formula as:
A * (9 х 3) + 0
B (9 х 2) + 2
C (9 х 3) + 2
D (9 х 2) + 0
E (9 х 3) + 3
On preparation appears a histological structure that is limited by the membrane which has a volume cytoplasm and plenty of
nucleuses. What name this structure has?
A *Symplast
B Syncytium
C Ground substance
D Cell
E Vacuoles
In the post-synthetic period of mitosis cycle was broken synthesis of proteins - tubulins that take part in formation of spindle of
division. It can result in violation:
A *Of divergence of chromosomes
B Of duration of mitosis
C Of spiralisation chromosomes
D Of cytokinesis
E Of antispiralisation chromosomes
On histological preparation painted by dye ferrous hematoxylin a dumbtellshaped cell is presented, in the poles of that visible
spiralisation of chromosomes. In what phase of cellular cycle is the cell?
A *In telophase
B In anaphase
C In metaphase
D In prophase
E In interphase
Scraping is done from the mucous membrane of cheek by means of spatula, a smear is made and painted by methylene blue.
Under a microscope near the internal membrane of nucleus of epithelial cells are visible the semispherical lumps of
heterochromatin. What is the structure?
A *Barr corpuscles
B Fordyce spots
C Buccal fat pad
D У-chromosome
E Ribosome
On an electronic photomicrograph is visible the fibroblast that products the components of intercellular substance. Define
organelles that take part in this process?
A * Rough Endoplasmic Reticulum and Golgi complex
B Smooth endoplasmic reticulum and Golgi complex
C Golgi complex and mitochondrion
D Golgi complex and lysosomes
E the Granular and smooth endoplasmic reticulum
Ultramicroscopic examination of “dark” hepatocytes population in the cell cytoplasm detected a developed granular
endoplasmic reticulum. What function has this organelle in these cells?
A *Synthesis of protein of blood plasma
B Synthesis of carbohydrates
C Detoxification
D Products of bile
E Depositing ions of calcium
From the cell was excluded Golgi complex by means of micromanipulator. How it will affect in later existence of cell?
A *Will be broken formation of lysosomes, maturation of secretory products of cell
B Will be broken by the process of mitosis
C Will be broken by the formation of ribosomes and synthesis of proteins
D Will develop autolysis, can lead cell to death
E Will be broken by processes of power exchange
EMBRYOLOGY
During gastrulation the Hensen's node remained underdeveloped in the embryo. Which axial organ will slow down its
development?
A *Chord
B Neural crests
C Neural groove
D Neural tube
E Mantle layer of the neural tube
Microspecimen analysis of child's finger skin revealed that epidermis has signs of inadequate development. What embryonal
leaf was damaged in the process of development?
A *Ectoderm
B Mesoderm
C Endoderm
D Mesenchyme
E Ectomezenchyme
During the experimental analysis of chondrohistogenesis a sclerotome was damaged. What cells will it make impossible to
differentiate?
A *Chondroblasts
B Smooth myocytes
C Myoblasts
D Fibroblasts
E Epidermocytes
Microspecimen of a child's finger skin reveals subnormal development of epidermis. What embryonic leaf was damaged in
course of development?
A *Ectoderm
B Mesoderm
C Endoderm
D Mesenchyme
E Ectomesenchyma
At a certain stage of development of a human embryo one can observe formation of a cavity in its structure, small light
blastomeres on the periphery and large dark blastomeres at one of the poles. The embryo at this stage of development is called:
A *Blastocyst
B Morula
C Zygote
D Gastrula
E Blastodisk
For an unknown reason the fertilization membrane of an embryo dissolved in the fallopian tube in the first critical period. What
complication of pregnancy is possible in this case?
A *Embryo implantation into the Fallopian tube
B Embryonic death
C Invagination of the blastocyst wall
D Return of blastocyst back to the ampullary portion of the tube
E Formation of two blastocysts
By producing a number of hormones placenta plays a part of temporary endocrine gland. What hormone may be detected in
woman's blood on the third or the forth day after begin of implantation, that is used in medicine for early pregnancy detection?
A *Chorionic gonadotropin
B Somatostatin
C Progesterone
D Vasopressin
E Oxytocin
Implantation process has two stages: adhesion and invasion. Morphological manifestation of blastocyte adhesion is:
A *Attachment of blastocyte to the endometrium
B Destruction of endometrium epithelium
C Destruction of connective tissue of endometrium
D Destruction of endometrium vessels
E Formation of lacunes
A newborn child has microcephalia. Doctors consider that this is the result of mother's taking actinomycin D during the
pregnancy. What embryonal leaf was influenced by this teratogen?
A *Ectoderma
B All leaves
C Entoderma
D Mesoderma
E Entoderma and mesoderma
Examination of uterine cavity revealed an embryonated ovum that wasn't attached to the endometrium. The embryo is in the
following stage of development:
A *Blastocyst
B Zygote
C Morula
D Gastrula
E Neurula
Examination of a pregnant woman who has been taking alcohol revealed diturbed anlage of ectoderma during the fetal life.
What derivatives of this leaf have defects?
A *Neural tube
B Kidneys
C Bowels epithelium
D Liver
E Sexual glands
Microscopic examination of oval shaped cell, with the size of 150 mkm, reveals cytoplasm with yolk, but doesn’t reveal
centrioles. What kind of cell is it?
A *Oocyte
B Leucocyte
C Myocyte
D Fibroblast
E Macrophage
In the cavity of uterus there was revealed the embryo which isn’t attached to endometrium. What stage of development is it?
A *Blastocysts
B Zygote
C Morula
D Neurula
E Gastrula
In the process of embryonic ectoderm differentiation a neural tubule, nervous combs, neurogenic placodes, skin ectoderm and
lamina prechordalis are developed. How is the process of forming a neural tubule defined?
A *Neurulation
B Gastrulation
C Somytogenesis
D Histogenesis
E Organogenesis
In preparation is visible an oocyte in the moment of impregnation it by spermatozoid. What is the main result of impregnation?
A *Formation zygote
B Determination sex of child
C Completion of meiosis an oocyte
D Penetration by spermatozoon an ovolema
E Cortical reaction
Implantation of embryo into the mucous membrane of uterus consists of two phases - adhesion and invasion. First phase
accompanied by:
A * Attaching blastocyst to the surface of endometrium
B Destruction of connecting tissue of endometrium
C Destruction of epitheliocytus of endometrium of uterus
D Activation of secretion of uterus glands
E Oppression of secretion of uterus glands
In a blastocyst which is, covered by tunicate impregnation, genetically inhibited the synthesis of lytic hormones in the cells of
the trophoblast. What process of embryogenesis can be done or can be delayed?
A * Implantation
B Delamination
C Immigration
D Gastrulation
E Epibolation
In preparation of main end of embryo, with the length of 6 mm, on the front surface of neck are visible piatenlike increases that
appeared due to proliferation of mesenchyme. How these formations of branchial apparatus are defined?
A *Branchial arches
B Branchial fissures
C Branchial pockets
D Branchial membranes
E Pharyngeal pockets
On the certain stage of ontogenesis of human is established during the certain stage of human ontogenesis. This function is
performed by the provisionally organ:
A *Placenta
B Yolk sac
C Amnion
D Umbilical cord
E Allantois
In preparation of main end of embryo of 5-th week formation are developments of branchial arcs. What form of first pair of
these formations?
A *Mandibularies and Maxillaries
B Mandibularies processus
C Maxillaries processus
D External acoustic duct
E Thyroid cartilage
In the first critical period in the fallopian tube without reason in an embryo was dissolution of fertilization membrane. What
complication of pregnancy is possible in this case?
A *Implantation of embryo in the wall of the uterus tube
B Death of embryo
C Invagination wall of blastocyst
D Return blastocyst back into the ampulla parts of uterus tube
E Formation of two blastocysts
One of critical periods of embryogenesis of human is implantation of embryo into wall of uterus during 7th days. What process
of gastrulation take place in the embryoplast in this period?
A *Delamination
B Migration
C Epibolation
D Invagination
E Neurulation
At microscopic research of internal genital woman organs that were remote during operation was found embryo built from two
blastomeres. Name the place of it localization on condition of normal development.
A * Fallopian tube (uterus tube) close to ampulla parts
B Fallopian tube, close to uterus part
C Cavity of uterus
D Abdominal region
E Ovary
During the gastrulation embryo goes through the hystotroph to the haematotroph methods of feeding. What pharmacist organ
does provide it first?
A *Chorion
B Trophoblast
C Yolk sack
D Amnion
E Allantois
The antigen of tissue compatibility of child inherits from a father and mother. It is known that expression of paternal antigens
in embryogenesis begins before time. But the immune system of mother doesn't tear away an embryo. What organ first of all is
prevent rejection an embryo of mother organism?
A *Chorion
B Amnion
C Allantois
D Yolk sack
E Umbilical Cord
On the micro preparation of human embryo, taken from involuntary abortion, is educed an embryonic corymb, in what
recognized two layers of the cell: ento- and ectoblast. In what stage of embryonic development was an embryo?
A *Gastrulation
B Progenesis
C Neurulation
D Histogenesis
E Organogenesis
During forensic examination of woman, which perished in an accident, an embryo was found on the stage of early gastrula.
Name the place of it localization on condition of it normal development.
A *Wall of uterus
B Ampulla part of tubes uterus
C Fallopian part of tubes uterus
D Ovary
E Abdominal region
Ecto- and entoderm are formed in the period of early gastrulation. In what mechanism these sheets appeared?
A *Delamination
B Invagination
C Epibolation
D Immigration
E Invagination, Epibolation
Spermatozoids move on woman genital tracts to side of ovule against liquid (distant stage of impregnation). What name has
this directed movement?
A *Rheotaxis
B Thermotaxis
C Chemotaxis
D Capacitaion
E Acrosome reaction
In embryogenesis of human in a 20-th day there is dissociating of embryo's body from pharmacist organs. In what way this
process is provided?
A *Truncal fold
B Amnion fold
C Celom
D Yolk sack
E Somits
In preparation of 10-daily embryo of human 2 bladders can be seen, that contact together (amniotic and vitelline). What is the
structure that lies in the place of their contact?
A *Embryonic corymb
B Bottom of amnion
C Roof of yolk sack
D Amniotic leg
E Fetal of mesoderm
It is known that some microorganisms that cause infectious diseases can pass through a placenta barrier. What structures are
included in its composition?
A *All structural components of third villi
B Chorion and amnion
C All structural components of secondary villi
D Allantois, yolk sack
E Basal lamina of endometrium with decidual cells
During histological preparation is determined a great number of mucous connective tissue (Wharton's jelly), vessel, and also
bits and pieces of vitelline stem, and allantois. What is the organ?
A *Umbilical cord
B Esophagus
C Ureter
D Urinary canal
E Vermicular appendix
Microscopic research of embryonic material in preparation is determined a yolk sack. What is the function of this organ for
human?
A *Hematopoesis
B Trophic
C Products of amniotic fluid.
D Excretory
E Protective
During formation of human embryo it is possible to see in his composition a cavity, light shallow blastomeres on periphery and
dark large blastomeres on one of poles. How is an embryo defined on this stage of development?
A *Blastocysts
B Morula
C Zygote
D Gastrula
E Embryo of disk
Microscopic research of embryo membranes is determined a chorion. What basic function is provided by this organ?
A *Metabolism between the organism of mother and fetus
B Hematopoesis
C Products of amniotic fluid
D Formation of primary gametes
E Formation of lymphocytes
In an embryo wasn’t formed primary Hensen’s node during gastrulation. The development of what axial organ will be
inhibiting?
A *Chorda
B Neural crest
C Neural groove
D Neural tube
E Mantle layer of the neural tube
The process of splitting of the zygote ends with the blastula formation. What type of blastula is typical for a human?
A *Blastocysts
B Coeloblastula
C Discoblastula
D Amphiblastula
E Morula
The gametes precursors (gonoblasts) were revealed in the embryo at 2nd-3rd weeks of embryogenesis. Where are these cells
differentiated?
A *In the yolk sac
B In the mesenchyme
C In the embryonic ectoderm
D In dermatomes
E In the embryonic endoderm
The myotome was destroyed in the rabbit embryo during the experiment. Disorder of what structure can be caused by this
manipulation?
A *Skeletal muscle
B Axial skeleton
C Skin connective tissue
D Smooth muscle
E Serous membrane
In the early stage of development (4 -10 week) in the main section of human embryo are found the bulges, covered by the thin
layer of epidermis, basis of which is formed by a mesenchyme. What the formation is it?
A The branchial arches
B The branchial fissures
C The branchial pockets
D The pallatin processus
E The frontal processus
As a result of the expression of individual components of genome, cells of embryo acquired characteristic morphological,
biochemical and functional features. What name this process has?
A *Differentiation
B Capacitation
C Reception
D Determination
E Induction
Endoscopic research in the cavity of uterus was found the human embryo, which was attached not fastened to endometrium.
What stage of embryogenesis is it?
A *"Free" blastocyst
B Zygote
C Morula
D Gastrula
E Neurula
Due to microscopic research of internal genital woman organs, that were remoted during surgery, there was found an embryo
that consists of 16 blastomeres covered by a specific membrane. How is this membrane named?
A *Membrane of fertilization
B Corona radiata
C Corona pollucida
D the Cellular membrane
E Oolemma
The 25-years-old woman visited a doctor – gynecologist abortion that happened on the 24th week of pregnancy. During the
detailed examination doctor set pathology in development of Chorion (child's part of placenta). What rudiment did undergone
changes?
A *Trophoblast
B Endoderm
C Ectoblast
D Epiblast
E Mesenchyme
What biological value has the process of impregnation that takes place during conferencing of male and female gametes?
A *The zygoid set of chromosomes recommences, determination of genetic embryo sex comes true, cleavage is initiated
B Formation of blastocysts
C Formation of Blastomeres
D Implantation is initiated
E Formation of Morula
The compression of umbilical cord of embryo is happened, but circulation of blood between an embryo and mother wasn't
disturbed. Presence of what structures contributed it first of all:
A *Mucous connective tissue
B Remain of allantois
C Layer of arteries
D Layer of vein
E Remain of vitelline stem
At an impregnation due to insufficiency of acrosome reaction dissolution of contacts wasn’t done between the follicle cells of
grainy zone of oocyte. What process was detected?
A *Denudation
B Capacitaion
C Cortical reaction
D Elimination
E Modification
A woman had infected by flu and it was appeared that it happened on 7-14 days of pregnancy. What consequences can appear?
A *Detection of formation of ecto- and endoderm
B Detection of formation of mesoderm
C Detection of formation of mesenchyme
D Detection of process of Epibolation
E Detection of process of invagination
A bounded vesicle with intestinal tube which is one of main organs is visible on micro section of human embryo. Primary
gametes and primary red corpuscles (megaloblasts) are located in his wall. Define, what is the main organ?
A. *Yolk sack
B. Allantois
C. Placenta
D. Umbilical cord
E. Amnion
The cut of organ is presented on histological preparation. Its basis is formed by a mucous connective tissue, two arteries and
vein. What organ is it?
A. *Umbilical cord
B. Allantois
C. Yolk sack
D. Amnion
E. Placenta
Outgrowth of ventral wall of primary bowel, which is growing in amniotic leg, is developed in the early stages of development
of human embryo. What is the name of this organ?
A. *Allantois
B. Yolk sack
C. Amnion
D. Placenta
E. Umbilical cord
In the conventional experiment fully inhibited the development of cells of mesenchyme. The disturbance development of what
muscular tissue will be observed?
A *Of the smooth muscular tissue of visceral type
B Of the muscular tissue of neural type
C Of the muscular tissue of epidermal type
D Of the cardiac muscular tissue
E Of the skeletal muscular tissue
Gastrulation is a period of embryogenesis, in what formed embryonic sheets, as a result of what an embryo acquires the threestratified structure. What mechanism of gastrulation is characteristic for the human embryo?
A *Delamination and immigration
B Invagination
C Epibolation
D Immigration and invagination
E Delamination and Epibolation
In normal condition circulation mothers blood and fetus doesn't have direct connections. Blood of fetus flows on the vessels of
Chorion villi, and mothers - circulates in between villi space endometrium of uterus. What shares mother blood from fetus
blood?
A *Haemochorion barrier
B Connective tissue partitions
C Fibrinoid of Langance
D the Amorphous fibrinoid of Рора
E the Locking plate of basic meatus of uterus
In slide is visible an oocyte in the moment of fertilization it by spermatozoid. Where does this process occur in normal
condition?
A *In ampulla part of fallopian tube
B In an abdominal cavity
C On the surface of ovary
D In the uterus
E In the isthmus of fallopian tube
A fetus membrane organ on the early stages of embryogenesis has a monolayer flat epithelium, which is from the third month
acquires a prismatic and cube form, participates in making of amniotic fluid. Name this organ:
A. *Amnion
B. Yolk sack
C. Allantois
D. Umbilical cord
E. Placenta
For diagnostics few micro slides are presented that are made of embryos age of two-eight weeks of embryo its development.
By what sign (beginning of period) is it possible to recognize this stage of embryo development on a micro slide?
A. *Formation of primary stripe
B. Cleavage
C. Gastrulation
D. Placentation
E. Neurulation
For diagnostics few micro slides are presented that represent the primary development period of human that lasts the first seven
days to embryogenesis. By what sign (beginning period) is it possible to recognize this stage of embryo development on a
micro slide
A. *Fertilization
B. Cleavage
C. Gastrulation
D. Placentation
E. Neurulation
Epithelial tissue
Study of a tubular organ revealed that its median membrane consists of solid hyaline rings. What epithelium lines mucous
membrane of this organ?
A *Multinuclear prismatic ciliated
B Monostratal prismatic glandular
C Monostratal prismatic with a border
D Multistratal squamous nonkeratinizing
E Monostratal cubical
Histological examination of a tissue sample revealed that the tissue had no blood vessels, and the cells were packed tightly
together making layers. Specify this tissue:
A *Epithelial
B Cartilaginous
C Osseous
D Nervous
E Muscular
In the experiment tight connections between the epitheliocytes were damaged. What function of the epithelium will be
disordered?
A. *Mechanical
B. Absorption
C. Vitamin D production
D. Secretory
E. Excretory
Histological investigation of the tissue revealed absence of blood vessels in it and tight connections between the cells with
formation of layers. What tissue is it?
A. *Epithelial
B. Cartilage
C. Bone
D. Nervous
E. Muscle
On the electron microphotograph of epithelial tissue under epitheliocytes damaging of the basal membrane were revealed.
What main function of the epithelium is disordered?
A. *Barrier
B. Absorption
C. Regeneration
D. Secretory
E. Exocytose
In a histological specimen the gland adenomeres should be determined. They are formed by the cells with central round
nucleus and basophilic cytoplasma. Determine the type of adenomeres:
A *Serous
B Mucous
C Combined
D Sebaceous
E Seromucous
Tunica intima of the vessel is lined with epithelium. Name it.
A. *Endothelium
B. Mesothelium
C. Epidermis
D. Transitional epithelium
E. Pseudostratified epithelium
In a histological specimen of the small intestine villi were determined. They covered with tissue which has only cells that rest
on the basal membrane. There are no blood vessels. What tissue covers villi?
A. *Epithelial tissue
B. Dense irregular connective tissue
C. Loose connective tissue
D. Smooth muscle
E. Reticular tissue
In the study of the epithelium revealed that it is composed of several layers of cells. Epithelial cells of the outer layer have no
nuclei. What is the kind of the epithelium?
A * Keratinizing stratified squamous epithelium
B Non-keratinizing stratified squamous epithelium
C Transitional
D Stratified columnar ciliated epithelium
E Stratified cuboidal epithelium
The rounded bodies that indicate that the cells were taken from the female mouth were revealed during the studies of the oral
cavity epithelial cells. What is the formation of a chromatin?
A *Barr bodies
B Lyon bodies
C Decondensed chromatin
D Euchromatin
E Pacinian corpuscles
Blood
Rejection of transplant developed in the patient after transplantation of heterogenous kidney. Call main effector cells, which
take place in this immune reaction?
A. *T killer
B. B lymphocytes
C. T suppressor
D. T helper
E. Plasma cells
Plasma cell produces specific antibody for specific antigen. During injection of antigen quantity of plasma cell increased. At
the expense of what blood cells occurred increasing of plasma cells quantity?
A. *B lymphocytes
B. Eosinophils
C. Basophils
D. T lymphocytes
E. Monocytes
During heterotransplantation of the organ was revealed rejection of transplant. What blood cells will ensure this process?
A. *T killer
B. T helper
C. T suppressor
D. T 0 lymphocytes
E. T memory cells
After investigation laboratory doctor has made an additional conclusion that blood belongs to a woman. Peculiarities of which
formed elements helped him to make this suggestion?
A. *Neutrophils
B. Erythrocytes
C. Lymphocytes
D. Monocytes
E. Basophils
Burn wound was closed by pig skin (heterotransplantation). Call effector cells which will reject transplant.
A. *T killer
B. T helper
C. T suppressor
D. B lymphocytes
E. Natural killer
In the blood of a patient with infection small amount of specific antibodies were revealed. Function of which cells of
connective tissue is depressed?
A. *Plasma cells
B. Lymphocytes
C. Macrophages
D. Neutrophils
E. Mast cells
The helminthiasis was detected at the child (10 years). What changes in the leukogram can be expected?
A *Number of eosinophils will increase
B Number of platelets will increase
C Number of red blood cells will increase
D Number of segmented neutrophils will increase
E Number of basophils will increase
The cells with the histamine and heparin granules in the cytoplasm were defined in the blood smear. What is the kind of the
cell?
A *Basophils
B Neutrophils
C Eosinophils
D Monocytes
E Erythrocytes
During investigation of the patient blood smear in the neutrophils were revealed bacteria. How did they appear their?
A. *Phagocytosis
B. Passive transport
C. Active transport
D. Excretion
E. Pinocytosis
During tooth extraction patient has longer bleeding from the wound compared with normal. Insufficient amount of what
formed elements of the blood caused increasing time of bleeding?
A. *Platelets
B. Erythrocytes
C. Monocytes
D. Lymphocytes
E. Neutrophils
6 years old child has hospital treatment with diagnosis allergic rhinitis. Changes in the leukocytic formula have been found.
Amount of what cells among leucocytes can be enlarged?
A. *Eosinophils
B. T lymphocytes
C. B lymphocytes
D. Basophils
E. Neutrophils
5 years old child has helminthes invasion, which caused organism sensibilization. What changes in the leukocytic formula can
be expected?
A *Increasing of the eosinophils number
B Increasing of the neutrophils number
C Reducing the eosinophils number
D Increasing of the monocytes number
E Increasing of the lymphocytes number
Inflammatory process in the tissues and organs is accompanied with hyperemia and edema. What connective tissue leucocytes
are causing these changes?
A *Basophils
B Eosinophils
C Neutrophils
D Lymphocytes
E Monocytes
During postembryonal haemopoiesis in the red bone marrow the cells of one of the cellular differons demonstrate a gradual
decrease in cytoplasm basophilia as well as an increase in oxyphilia, the nucleus is being forced out. Such morphological
changes are typical for the following haemopoiesis type:
A *Erythropoiesis
B Lymphopoiesis
C Neutrophil cytopoiesis
D Eosinophil cytopoiesis
E Basophil cytopoiesis
Filopodia of megakaryocytes enter lumen of the vessels through sinusoidal capillaries fenestrae of red bone marrow and
become fragmented in smaller parts. What formed elements of the blood will be formed in this way?
A. *Platelets
B. Erythrocytes
C. Lymphocytes
D. Reticulocytes
E. Monocytes
Blood analysis of a 16-year-old girl suffering from the autoimmune inflammation of thyroid gland revealed multiple plasmatic
cells. Such increase in plasmocyte number is caused by proliferation and differentiation of the following blood cells:
A *B-lymphocytes
B T-helpers
C Tissue basophils
D T-killers
E T-supressors
In the experiment ribosomes of the poly chromatophil erythroblasts were destroyed in the human red bone marrow. Synthesis
of what specific protein will be disordered?
A. *Globin
B. Fibrinogen
C. Collagen
D. Elastin
E. Laminine
The rounded cells with the segmented nuclei are predominating from the leukocytes in the smear of peripheral blood. The
granules of their cytoplasm stained both acidic and basic dyes. What are these cells?
A *Segmented neutrophils
B Basophils
C Eosinophils
D Young neutrophils
E Monocytes
The large cells with low-basophilic cytoplasm and bean-shaped nucleus were founded in a blood smear. The cell is the largest
from visible in the visual field. What are the cells?
A *Monocytes
B Macrophages
C Plasmocytes
D Middle lymphocytes
E Small lymphocytes
The cells of granulocytic series were revealed at the biopsy material of red bone marrow. Specify what changes occur in the
nucleus during the differentiation of these cells?
A *Segmentation
B Polyploidisation
C Pyknosis
D Enucleation
E Increasing the size
The cells, which accounts for 0,5% of the total leukocytes number with the S-shaped curved nucleus and metachromatic
colored granules in the cytoplasm, were founded in the patient's blood smear. What are these cells?
A *Basophils
B Neutrophils
C Eosinophils
D Monocytes
E Lymphocytes
In the red bone marrow specimen conglomerates of giant cells in tight connections with sinusoidal capillaries are revealed. Call
formed elements of the blood which will be formed from these cells.
A. *Platelets
B. Erythrocytes
C. Leucocytes
D. Monocytes
E. Lymphocytes
Connective tissue
A specimen of connective tissue of derma was stained with Sudan III and hematoxylin. There are clusters of big polygonal
cells that turned orange. Their nuclei are flattened and located on periphery. What tissue is it?
A *White adipose
B Brown adipose
C Reticular connective
D Hyaline cartilaginous
E Lamellar osseous
When a wound heals, a scar takes its place. What substance is the main component of its connective tissue?
A *Collagen
B Elastin
C Keratan sulfate
D Chondroitin sulfate
E Hyaluronic acid
A chemical burn of esophagus caused it's local constriction as a result of scar formation. What cells of loose connective tissue
take part in scar formation?
A *Mature specialized fibroblasts
B Immature nonspecialized fibroblasts
C Fibrocytes
D Miofibroblasts
E Fibroclasts
In loose connective tissue of the salivary glands revealed oval shape middle size cells which synthesized antibodies. Spherical
eccentrically positioned nucleus with chromatin clumps of which resemble cartwheel or clock face. Call these cells.
A. *Plasma cells
B. Adipose cells
C. Neutrophils
D. Fibroblasts
E. Macrophages
Chronicle inflammation of the gingiva is finished with excessive excrescence of connective tissue fibers. What cells took
active place in this process?
A. *Fibroblasts
B. Osteoblasts
C. Fibrocytes
D. Macrophages
E. Osteoclasts
A foreign body entered into the skin and leads to inflammation development. What connective tissue cells take place against
the foreign body in the skin reaction?
A *Neutrophils, macrophages, fibroblasts
B Macrophages
C Melanocytes
D Adipocytes
E Adventitial cells
During investigation of connective tissue slides neutrophils are revealed. What function do these cells have when they leave
blood and enter tissue?
A. *Microorganisms phagocytosis
B. Trophic
C. Support
D. Regulation of smooth muscle cells contraction
E. Expand blood vessels
Decreased blood supply of the organs causes hypoxia that activates fibroblasts function. Volume of what elements does
increase in this case?
A *Intercellular substance
B Vessels of microvascular bed
C Nerve elements
D Parenchymatous elements of the organ
E Lymphatic vessels
The damaged Achilles tendon function was renewed after the treating. What is the mechanism of tendon regeneration?
A *Synthesis of the collagen fibers
B Synthesis of the hyaline cartilage
C Formation of the adipose tissue
D Synthesis of the fibrous cartilage
E Replacements of the place of injury by the muscle tissue
A patient underwent Caesarean section. During the operation a long incision was made in the uterus wall and the fetus was
extracted from uterus. Healing of the sutured myometrium will proceed in the following way:
A *Formation of a fibrous cicatrix
B Formation of smooth muscular tissue
C Formation of cross-striated muscle fibers
D Proliferation of myosatellitocytes
E Hypertrophy of smooth myocytes
The lower limb was injured during the athlete training. Traumatologist made the diagnosis: the tendon rupture. What type of
connective tissue the tendon belongs to?
A *Dense regular fibrous tissue
B Dense irregular fibrous tissue
C Loose connective tissue
D Reticular tissue
E Cartilage tissue
In loose connective tissue histological specimen revealed relatively big cells with basophilic metachromatic granularity. These
granules include histamine and heparin. Call these cells.
A. *Mast cells
B. Fibroblasts
C. Macrophages
D. Plasma cells
E. Adipose cells
Connective tissue consists of parallel collagen fibers separated by fibroblasts. What is the name of this type of connective
tissue?
A. *Dense regular connective tissue
B. Loose connective tissue
C. Reticular connective tissue
D. Dense irregular connective tissue
E. Mucous connective tissue
Cartilage
As a result of a chest trauma the costal cartilage was damaged. The cartilage regenerates due to the following layer of
perichondrium:
A *Chondrogenic
B Fibrous
C Elastic
D Collagen
E Sharpey's fibers
Regeneration of articular cartilage of the mandible after damage is a great problem for traumotologists and dentists. What is
the reason of limited hyaline cartilage regeneration of the bone articular surface?
A. *Absence of the perichondrium
B. Absence of isogenic groups
C. Presence of the vessels
D. Huge mechanic loading
E. Large content of nonorganic compounds
In the histological connective tissue specimen that stained with hematoxylin eosin revealed isogenic groups of cells surrounded
by ground substance. Fibrous structures are not visible. What tissue is represented in the slide?
A *Hyaline cartilage
B Elastic cartilage
C Dense regular connective tissue
D Fibrous cartilage
E Bone tissue
In the result of chondrodisplasia (anomaly of cartilage development) fiber cartilage was damaged. Where pathological changes
can be seen?
A. *In the intervertebral discs
B. In the auricle
C. In the trachea
D. In the pharynx
E. In the bronchi
The two slides were proposed to the student. At the 1 st one there is the elastic cartilage (stained by the orcein), at the 2 nd one –
the hyaline cartilage (stained by the hematoxylin-eosin). According to what features can we tell one from the other?
A *Presence of the elastic fibers
B Presence of the cells isogenic groups
C Presence of young cartilage zone
D Presence of the perichondrium
E Presence of amorphous substance
The articular cartilage, as it is known, doesn’t have the perichondrium. What kind of growth is possible at this cartilage during
the regeneration process?
A *Interstitial
B Appositional
C By application
D Appositional and interstitial
E It is not growing
During the experimental analysis of chondrohistogenesis a sclerotome was damaged. What cells will it make impossible to
differentiate?
A *Chondroblasts
B Smooth myocytes
C Myoblasts
D Fibroblasts
E Epidermocytes
The isogenic cell groups were revealed at the histological slide of the cartilage. What cells are the primary in the formation of
these groups?
A *Chondrocytes I type.
B Chondroblasts
C Prechondroblasts
D Chondrocytes II type
E Chondrocytes III type
Bone tissue
The symptoms of regeneration process (callus) on the place of fracture were revealed in the histologic specimen of tubular
bone. What tissue forms this structure?
A *Fibrous bone tissue
B Loose connective tissue
C Reticular tissue
D Epithelial tissue
E Lamellar bone tissue
Examination of a histological specimen of tubular bone revealed signs of regeneration process (callus). What tissue is this
structured formed of?
A *Rough fibrous osseous
B Loose connective
C Reticular
D Epithelial
E Lamellar osseous
A histological specimen presents the tissue that contains cells having no processes and a few tens of nuclei each. One of cell
surfaces has a corrugated zone that provides secretion of hydrolytic elements. What tissue is it?
A *Osseous tissue
B Cartilaginous tissue
C Epithelial tissue
D Nerve tissue
E Muscular tissue
Calcification of the intercellular substance of bone tissue is accompanied by the deposition of hydroxyapatite crystals along the
collagen fibers. This process requires the presence of alkaline phosphatase in the intercellular substance. What cell produces
this enzyme?
A *Osteoblast
B Osteocyte
C Osteoclast
D Chondroblast
E Chondrocyte
Calcification of the intercellular substance of bone tissue is accompanied by the deposition of hydroxyapatite crystals along the
collagen fibers. This process requires the presence of alkaline phosphatase in the intercellular substance. What cell produces
this enzyme?
A *Osteoblasts
B Osteocytes
C Osteoclasts
D Chondroblasts
E Chondrocytes
There are histogenetic cells row among bone tissue cells. What cells don’t constitute differon?
A. *Osteoclasts
B. Stem cells
C. Osteocytes
D. Osteoblasts
E. Osseus stem cells
There are two ways of bone development during histogenesis. What period doesn’t typical for direct osteogenesis?
A. *Formation of epiphyseal center of ossification
B. Formation in the mesenchyme precursor of the bone
C. Osteoid period
D. Formation of spongy bone tissue
E. Replacement of spongy bone tissue by compact bone tissue
In course of indirect histogenesis of tubular bone tissue a plate is formed between epiphyseal and diaphyseal ossification a
center that provides further lengthwise growth of bones. What structure is it?
A *Metaphyseal plate
B Osseous cuff
C Osseous plate
D Osteon
E Layer of interior general plates
In the bone tissue multinucleated large cells with numerous lysosomes and ruffled border were revealed. Call these cells.
A. *Osteoclasts
B. Mesenchymal cells
C. Stem cells
D. Osteoblasts
E. Osteocytes
The excessive loss of bone tissue mass (it reflects the development of osteoporosis) is observed at the elderly people.
Activation of whst bone cells can cause the development of this disease?
A *Osteoclast
B Osteoblasts
C Macrophages
D Tissue basophils
E Osteocytes
A histological specimen presents the tissue that contains cells having no processes and a few tens of nuclei each. One of cell
surfaces has a corrugated zone that provides secretion of hydrolytic elements. What tissue is it?
A *Osseous tissue
B Cartilaginous tissue
C Epithelial tissue
D Nerve tissue
E Muscular tissue
Muscle tissue
The regeneration process of damaged skeletal muscles іs very slow. What elements of musculoskeletal fiber take part in the
process of regeneration?
A *Myosatellitocytes
B Myoblasts
C Smooth myocytes
D Myofibroblasts
E Myoepithelial cells
Striated skeletal muscle tissue has all characteristics which were listed further except:
A. *Cells structure
B. Contractive ability
C. Presence of satellite cells
D. Presence of motor plaque
E. Presence of connective tissue layer
Long time mice were swimming in the pool. During morphological investigation of their skeletal muscles were revealed
increasing quantity of the mitochondria with cristae and light matrix. What cell function will be found in the stressful
condition?
A. *Energetic
B. Secretory
C. Synthetic
D. Protective
E. Transport
During histological investigation of the cheek biopsy was found that it formed by skeletal muscle tissue. Call structural and
functional unit of this tissue.
A. *Muscle fiber
B. Cardiac muscle cell
C. Smooth muscle cell
D. Myosatellite cells
E. Myofibril
In the histological specimen are revealed sheaths of the loose connective tissue that immediately surrounds individual muscle
fibers and intertwined with muscle fibers sarcolemma. What histological structure has been revealed?
A. *Endomysium
B. Capillaries
C. Nervous ending
D. Actin fiber
E. Myosin fiber
Base of the cheek constitutes by cheek muscle which is derivative of second visceral arch. What type of muscle tissue is it?
A. *Somatic
B. Coelomic
C. Visceral
D. Neural
E. Epidermal
In one of the layers of the hollow organ anastomosing fibers with nucleus are revealed. Fibers consist of cells which in the
connection regions forms intercalated disks. Which tissue constitutes this layer?
A. *Cardiac muscle tissue
B. Skeletal muscle tissue
C. Smooth muscle tissue
D. Dense regular connective tissue
E. Dense irregular connective tissue
The destruction of the thick myofilaments is observed after mechanical injury of striated muscle fibers. Where is the
localization of the pathological changes?
A *In the disk A
B In the disk I
C In the disk A half
D In the A and I disks
E In the disk I half
Destruction of thin microfilaments is observed during the research of a striated muscle fiber after the action of hydrolytic
ferments. Which structures have been damaged?
A. *Actin myofilaments
B. Myosin myofilaments
C. Tonofibrils
D. Tropocollagen complex
E. Sarcoplasmic reticulum
Myotom was destroyed in the rabbit embryo during experiment. Which structure development disorder has been caused by this
manipulation?
A. *Skeletal muscle
B. Skeleton
C. Skin connective tissue
D. Smooth muscle
E. Serous layers
Fibers were presented in the histological specimen of the stomach pyloric region. They include spindle shape cells with rod
like nucleus located in the center. What tissue do these cells form?
A. *Smooth muscle
B. Loose connective tissue
C. Skeletal muscle
D. Epithelial
E. Reticular
It is known, that calcium ions along with other factors provide muscle tissue contraction. With what structures does calcium
interact during contraction?
A. *Troponin protein of thin fibrils
B. Myosin protein of thick fibrils
C. Actin protein of thin fibrils
D. Actomyosin sarcolemma complex
E. Calsequestrin
A microspecimen of the submandibular salivary gland shows some basket-shaped cells concentrated around the acines and
excretory ducts. These cells surround bases of the serous cells and are called myoepitheliocytes. These cells relate to the
following tissue:
A *Muscular tissue
B Epithelial tissue
C Neural tissue
D Special connective tissue
E Loose fibrous connective tissue
The regeneration process of damaged skeletal muscles іs very slow. What elements of musculoskeletal fiber take part in the
process of regeneration?
A *Myosatellitocytes
B Myoblasts
C Smooth myocytes
D Myofibroblasts
E Myoepithelial cells
Nervous tissue
A sensory nerve ganglion consists of roundish neurocytes with one process that divides into axon and dendrite at a certain
distance from perikaryon. What are such cells called?
A *Pseudounipolar
B Unipolar
C Bipolar
D Multipolar
E Apolar
Damaged nervous fibers capable regenerate their integrity. What neuroglia cells take active place in it?
A. *Olygodendrocytes
B. Microglia
C. Ependymal cells
D. Fibrous astrocytes
E. Protoplasmic astrocytes
In the histological specimen of the cerebellum transverse section in its gray matter observed large amount of multipolar
neurons. Due to which morphological peculiarity these cells can be classified as multipolar?
A. *Amount of processes
B. Length]of processes
C. Shape of terminal dilation of axon
D. Perikaryon shape
E. Cells shape
Choose one incorrect answer. Myelinated fibers have all character signs except:
A. *Several axons
B. One axon
C. Node of Ranvier
D. Neurofilaments
E. Neurolemmacytes
During a conditional experiment the action of toxic substance enhances the mechanism of nerve impulse transfer. What
structure provides this function?
A. *Synapse
B. Neurolemma
C. Neurofibril
D. Mitochondrion
E. Nissl’s body
Central nervous system
A ventral root of spinal cord was damaged as a result of a trauma. The following processes of the following neurons were
damaged:
A *Axons of motor neurons
B Dendrites of motor neurons
C Axons of sensory neurons
D Dendrites of sensory neurons
E Dendrites of internuncial neurons
One of sections of central nervous system has layerwise arrangement of neurocytes. Among them there are cells of the
following forms: stellate, fusiform, horizontal, pyramidal. What section of central nervous system is this structure typical for?
A *Cortex of cerebrum
B Spinal cord
C Cerebellum
D Medulla oblongata
E Hypothalamus
As result of a trauma, a patient has damaged frontal spinal roots. What structures are likely to be affected?
A *Axons of the motoneurons and axons of the lateral horn neurons
B Central processes of the sensory neurons of the spinal ganglions
C Peripheral processes of the sensory neurons of the spinal ganglions
D Axons of the lateral horn neurons
E Dendrites of the spinal ganglion neurons
A patient had a trauma that led to the injury of front spinal roots. Denote the damaged structures:
A *Axons of motoneurons and lateral horn neurons
B Central processes of spinal ganglion neurons
C Peripheral processes of spinal ganglion neurons
D Axons of lateral horn neurons
E Axons of motoneurons
In a specimen that was coloured by method of silver impregnation some piriform cells with 2-3 evident dendrites were found.
What structure is being analysed?
A *Cerebellar cortex
B Spiral organ of middle ear
C Retina
D Cerebral cortex
E Spinal ganglion
In the newborn child tumor of the spinal cord gray matter was found. What embryonal germ is connected with this?
A. *Mantle zone
B. Ependymal zone
C. Nervous cord
D. Ganglionar plate
E. Mesoderm
As result of a trauma, a patient has damaged posterior roots of the spinal cord. What structures have been affected?
A. *Sensory neurons
B. Motor neurons
C. Interneurons
D. Dendrites of neurons of spinal ganglions
E. Central processes of sensitive neurons of spinal ganglions
In the microspecimen of the cerebral cortex large pyramidal cells are reveled. These cells are most significant sign of the
cerebral cortex. Discover of these cells is associated with next name:
A. *Betz
B. Goldgi
C. Lenoshek
D. Nissl
E. Kakhal
In the histological specimen was represented precentral gyrus of the cerebral cortex. Indicate which layers development best in
this layer?
A. *Pyramidal, ganglionar and polymorphic cells layer (external and internal pyramidal and polymorphic cells layer)
B. Molecular
C. External and internal granular
D. Molecular and polymorphic cells layer
E. Molecular, pyramidal and ganglionar (external and internal pyramidal, molecular)
Peripheral nervous system
During surgical operation nerve was cut. After some period of time it regenerated. Call cell which provide its regeneration.
A. *Neurolemmocytes
B. Astrocytes
C. Ependymal cells
D. Microglia cells
E. Mantle glyocytes
Microscopic analysis of a specimen revealed an organ of nervous system that consists of pseudounipolar neurons covered with
glial and connective tissue membranes. Determine this organ:
A *Spinal ganglion
B Vegetative ganglion
C Spinal cord
D Cerebellum
E Cortex of cerebrum
Sensory organs. Eye
As result of punctate retinal hemorrhage a patient lost ability to see objects in the centre of visual field. In what part of retina
did the hemorrhage take place?
A *Yellow spot
B Ciliary part of retina
C Iris
D Blind spot
E Vascular membrane
The deficit of vitamin A causes the disorder of twilight vision. What cells is the photoreceptor function typical for?
A *Rod neurosensory cells
B Horizontal neurocytes
C Conic neurosensory cells
D Bipolar neurons
E Ganglionic nerve cells
A histological specimen of an eyeball shows a structure in form of a convexoconvex formation connected with the ciliary body
by the fibers of ciliary zonule and covered with a transparent capsule. Specify this structure:
A *Crystalline lens
B Vitreous body
C Ciliary body
D Cornea
E Sclera
An infectious disease caused contractive activity of muscles that contract and dilate eye pupil (paralytic state). What functional
eye system was damaged?
A *Accomodative
B Dioptric
C Ancillary
D Photosensory
E Lacrimal apparatus
The ciliary body of a patient is damaged. Function of what eye apparatus suffers>
A. *Accommodation
B. Light- conductive
C. Photosensory
D. Protective
E. Trophic
A lot of people with age have clouding of the lens (phacoscotasmus or cataract) that leading to the partial or total blindness.
What optical and chemical properties of the lens fibers protein will be disordered?
A. *Crystallin
B. Viterin
C. Dinein
D. Rhodopsin
E. Iodopsin
In a histological specimen a structure of eyeball wall is detected; blood vessels are absent in this structure. What structure is
characterized by this morphologic sign?
A. *Cornea
B. Ciliary body
C. Choroid
D. Iris
E. Retina
In the microspecimen of the posterior eye wall was revealed undeveloped retinal pigment layer. What layer of the eye ball was
injured during development process?
A. *External
B. Medial
C. Internal
D. Ganglion cells
E. Mesenchyme
During investigation of the ocular fundus ophthalmologist drew attention at the fovea centralis of the retina. What fovea is
this?
A. *Area of greatest visual acuity
B. Blind spot
C. Place of optic nerve exit
D. Place where retina’s veins exit eye
E. Place without photoreceptors cells
Patient has clouding of the lens or cataract. What structure of the lens will be primary disordered in this situation?
A. *Lens fibers
B. Lens epithelium
C. Lens core
D. Lens capsule
E. Ciliary fibers
In a histological specimen of an eyeball a biconvex structure is connected with the ciliary body by means of the ciliary zonulae
fibers and is covered with a transparent capsule from above. Name this structure:
A *Crystalline lens
B Vitreous body
C Ciliary body
D Cornea
E Sclera
Teenager consulted a doctor with disorder of visual acuity. Doctor explained that this problem connected with accommodation
spasm. Which of mentioned components of the eye ball constitute eye accommodation apparatus?
A. *Ciliary muscle
B. Retina
C. Vitreous humor
D. Sclera
E. Cornea
Condition of the macula lutea was studied during ophthalmological investigation of the ocular fundus. What retina cells
elements are most found in the macula lutea?
A. *Cones
B. Horizontal neurons
C. Pigment cells
D. Rods
E. Ganglion cells
In the electron microphotograph of the sense organ revealed cells peripheral part of which constitutes from two segments. The
outer segment has membrane half discs and the inner one has ellipsoid. In what organ does this structure locate?
A. *In the organ of vision
B. In the organ of taste
C. In the organ of smell
D. In the vestibular organ
E. In the auditory organ
The increased intraocular tension is observed in the patient with glaucoma. Secretion of aqueous humor by the ciliary body is
normal. Injury of what structure of the eyeball wall caused the disorder of flow-out from the anterior chamber?
A *Venous sinus
B Ciliary body
C Choroid
D Ciliary muscle
E Back epithelium of cornea
In the electron microphotograph revealed cell of neural origin. Terminal part of the cell dendrite has cylindrical shape and
consists from 1000 enclosed membrane discs. What cell is this?
A. *Cone cell
B. Ventral horn of the spinal cord
C. Sensory ganglia neuron
D. Cerebral cortex neuron
E. Rod cell
Sensory organs. Ear
As result of head trauma, a 32 year old man has damaged ampullas of semicircular ducts.
be disturbed?
A *Angular acceleration
B Vibration
C Gravitation
D Linear acceleration
E Vibration and gravitation
What stimuli perception will
A histological specimen presents a receptor zone of a sensoepithelial sense organ. Cells of this zone are placed upon the basal
membrane and include the following types: external and internal receptor cells, external and internal phalangeal cell, stem
cells, external limiting cells and external supporting cell. The described receptor zone belongs to the following sense organ:
A *Acoustic organ
B Visual organ
C Gustatory organ
D Equilibrium organ
E Olfactory organ
After having a virus disease patient has lost ability to hear. What cells of the Corti organ (spiral organ) had been damaged?
A. *Receptor hair cells
B. Phalangeal cells
C. Outer supporting cells
D. Inner supporting cells
E. Pillar cells
During an experiment the median part of an animal's cochlea was damaged. This resulted in impaired perception of acoustic
vibrations of the following frequency:
A *Medium
B Low
C High
D High and medium
E Low and medium
Skin.
Histological study of a microslide of human skin found only dense irregular connective tissue. Which layer of this organ was
analysed?
A *Reticular dermis
B Papillary dermis
C Subcutaneous adipose tissue
D Epidermis
E Basal layer of epidermis
An embryon has signs of disturbed process of dorsal mesoderm segmentation and somite generation. What part of skin is most
likely to have developmental abnormalities?
A *Derma
B Hair
C Sebaceous glands
D Epidermis
E Sudoriferous glands
What cells in the skin epidermis together with afferent fibers terminals form tactile receptors?
A. *Merkel cells
B. Melanocytes
C. Epidermal cells of stratum basale
D. Stratum spinosum cells
E. Langerhans cell
Human skin is very strong for the rupture. It is known that skin consists of epithelial tissue and two types of the connective
tissue. Which from mentioned below tissues provide tensile strength of the skin?
A. *Dense irregular connective tissue
B. Stratified squamous epithelium
C. Loose connective tissue
D. Simple epithelium
E. Transitional epithelium
After the skin burn stratum basale cells were suffered. What function of the epidermis will be weakening or depressed first of
all?
A. *Regenerative
B. Protective
C. Absorptive
D. Barrier
E. Dielectric
After ultraviolet radiation over time skin darkens. Synthesis of what substance will be activated in the melanocytes after
ultraviolet radiation.
A. *Melanin
B. Lipids
C. Eleidin
D. Keratin
E. Kertogyalin
Under the radiation influence epidermal cells of the stratum basale were damaged. What function of the epidermis will be
weakening or depressed first of all?
A. *Regenerative
B. Protective
C. Absorptive
D. Barrier
E. Dielectric
The reticular layer of the skin is damaged after trauma. This layer will recover with the help of a cell population. Name this
cell population.
A. *Fibroblast
B. Macrophages
C. Lymphoblastic
D. Mast cells
E. Plasma cells
Cells with processes and dark brown granules in the cytoplasm were revealed in the skin epidermis of the biopsy material.
What cell are these?
A. *Melanocytes
B. Intraepidermal macrophages
C. Keratinocytes
D. Merkel cells
E. Lymphocytes
With age wrinkles and plications are appeared in the human skin. Changes in which skin structures mainly caused this
condition.
A. *In the elastic fibers
B. In the collagen fibers
C. In the epidermis
D. In the ground substance
E. In the subcutaneous adipose tissue
Histological study of a microspecimen of human skin found only dense irregular connective tissue. Which layer of this organ
was analyzed?
A *Reticular dermis
B Papillary dermis
C Subcutaneous adipose tissue
D Epidermis
E Basal layer of epidermis
There is histological specimen of the skin epidermis bioptic sample taken from the healthy adult. Cells division can be seen in
the stratum basale. What process provide these cells.
A. *Physiologic regeneration
B. Differentiation
C. Adaptation
D. Reparation
E. Apoptosis
On the body human surface distinguishes areas of thick and thin skin. What skin layer has different structure in these areas?
A. *Epidermis
B. Papillary layer
C. Reticular layer
D. Derma
E. Hypodermis
There are cells in the epidermis which have protective function and have mononuclear genesis. What cells are these?
A. *Langerhans cells
B. Melanocytes
C. Keratinocytes of stratum basale
D. Keratinocytes of stratum spinosum
E. Keratinocytes of stratum granulosum
Cardiovascular system. Arteries. Microcirculatory vessels.
A histological specimen shows a blood vessel. Its inner coat is composed by endothelium, subendothelium and internal elastic
membrane. The middle coat is enriched with smooth myocytes. Such morphological characteristics are typical for the
following vessel:
A *Muscular-type artery
B Elastic-type artery
C Capillary
D Non-muscular vein
E Muscular-type vein
On the electron micrograph of capillary are revealed fenestrae between endothelial cells and partial or total absence of basal
lamina underlying the endothelium. What type of capillary is it?
A. *Sinusoidal
B. Somatic
C. Visceral
D. Atypical
E. Shunt
Arterioles play an important role in the blood supply of the organs functional units. Which of these structures perform this
function?
A *Myocytes
B External elastic membrane
C Internal elastic membrane
D Special connective tissue cells
E Endothelial
From 40 to 60 fenestrated elastic membranes were founded at the histological preparations stained by the orcein. Name this
vessel.
A *Artery of elastic type
B Artery of muscular type
C Artery of mixed type
D Vein of muscular type
E Vein of unmuscular type
The contraction of the smooth muscle cells of arterioles occurs after the adrenalin releasing from the adrenal medulla into the
blood. What are the features of these vessels structure?
A * Availability of perforations in the endothelium basement membrane and the internal elastic membrane
B Single position of the smooth muscle cells
C Presence of the effector endings on the pericytes
D Presence of the contacts between the pericytes
E Availability of precapillary sphincters
On a electron micrograph of a tunica intima fragment revealed cells which rests on the basal membrane and connected with
each other by means of desmosomes and zonulae occludentes. Name these cells.
A. *Endothelium
B. Mesothelium
C. Epidermis
D. Epithelioreticular cells
E. Macrophages
In the slide of the microvascular bed vessel tunica media consists of 1-2 layers of smooth muscle cells which have spiral
orientation. Tunica adventitia is a thin, sheath of loose connective tissue. What vessel is it?
A. *Arteriole
B. Venule
C. Capillary
D. Postcapillary
E. Arterio-venous shunt
Obliterating atherosclerosis causes changes in the vessels of the lower extremities. A histological specimen of such a vessel
evidently presents both internal and external elastic membranes; middle membrane contains a lot of smooth muscle cells. What
vessel is affected in case of this disease?
A
B
C
D
E
*Artery of muscular type
Artery of elastic type
Artery of mixed type
Vein with strongly developed muscles
Lymph node
A histological specimen presents an artery. One of the membranes of its wall has flat cells lying on the basal membrane. What
type of cells is it?
A *Endotheliocytes
B Mesotheliocytes
C Smooth muscle cells
D Fibroblasts
E Macrophages
Wall of the vessels often have huge morphological difference in the structure of the tunica media. What is the reason of
appearance such specific peculiarities in the structure of this tunica in different vessels?
A. *Hemodynamic condition
B. Influence of endocrine system organs
C. Regulation from central nervous system
D. Inductive influence of vegetative ganglion neurons
E. Large content of catecholamines in the blood
Large arteries during systole stretch out and return in previous condition during diastole providing stability of bloodstream.
What elements of vessel wall will explain this?
A. *Elastic fibers
B. Muscle fibers
C. Reticular fibers
D. Collagen fibers
E. Large amount of fibroblasts
In the vessel slide one found prominent internal and external elastic membranes and a lot of smooth muscle cells in the tunica
media. What type of vessel is it?
A. *Muscular artery
B. Small artery (mixed)
C. Large vein (muscular)
D. Elastic artery
E. Extraorganic lymphatic system
Intralobular capillaries of a liver specimen have wide irregular lumen. Basal membrane is absent in the major part of the
capillary. What type of capillaries is it?
A *Sinusoid
B Visceral
C Somatic
D Precapillaries
E Postcapillaries
Tunica intima of a vessel is lined with epithelium from within. What epithelium is this?
A. *Endothelium
B. Mesothelium
C. Epidermis
D. Transitional epithelium
E. Pseudostratified epithelium
Obliterating atherosclerosis causes changes in the vessels of the lower extermities. A histological specimen of such a vessel
evidently presents both internal and external elastic membranes, middle membrane contains a lot of myocytes. What vessel is
affected in case of this disease?
A *Artery of muscular type
B Artery of elastic type
C Artery of mixed type
D Vein with strongly developed muscles
E Lymph node
A histological specimen shows a blood vessel. Its inner coat is composed by endothelium, subendothelium and internal elastic
membrane. The middle coat is enriched with smooth myocytes. Such morphological characteristics are typical for the
following vessel:
A *Muscular-type artery
B Elastic-type artery
C Capillary
D Non-muscular vein
E Muscular-type vein
During investigation of skin bioptat in the derma revealed vessels which have thick layer of smooth muscle cells in the tunica
media. What is the name of these vessels?
A. *Muscular artery
B. Capillaries
C. Arterioles
D. Venules
E. Arterio-venous shunts
In a histological specimen of tubular organ dyed with orcein have been detected about 50 thick membranes which have wiggly
appearance and formed tunica media of this organ. Name this organ.
A. *Aorta
B. Muscular artery
C. Esophagus
D. Trachea
E. Heart wall
In a histological specimen one can see an artery. There are flat cells on the basal membrane in one of the tunics of its wall.
Name these cells.
A. *Endothelium
B. Mesothelium
C. Smooth muscle cells
D. Fibroblasts
E. Macrophages
In the microspecimen of red bone marrow there were revealed multiple capillaries through the walls of which mature blood
cells penetrated. What type of capillaries is it?
A *Sinusoidal
B Fenestrated
C Somatic
D Visceral
E Lymphatic
Cardiovascular system. Veins. Lymphatic vessels.
Morphological examination revealed in histological specimen of biopsy material an irregular-shaped vessel. Its middle
membrane is formed by bundles of smooth myocytes and layers of connective tissue. What type of vessel is it?
A *Vein of muscular type
B Artery uf muscular type
C Lymphatic vessel
D Venule
E Arteriole
A specimen of the pia mater shows a vessel with no middle membrane in its wall, its outer membrane adheres to the
surrounding tissues, the inner membrane is made up of the basal membrane and endothelium. Specify this vessel:
A *Fibrous vein
B Muscular vein with weakly developed muscular elements
C Muscular artery
D Arteriola
E Mixed artery
Histological specimen presents a vessel the wall of which consists of endothelium, basal membrane and loose connective
tissue. What type of vessel is it?
A *Vein of non-muscular type
B Artery
C Vein of muscular type
D Blood capillary
E Lymphatic capillary
A histological specimen of spleen shows a vessel with a wall consisting of endothelium and subendothelial layer, median
membrane is absent, exterior membrane inosculates with the layers of spleen connective tissue. What vessel is it?
A *Vein of non-muscular type
B Vein of muscular type
C Artery of muscular type
D Arteriole
E Capillary
A specimen of pia mater includes a vessel whose wall doesn't have the tunica media, the tunica externa is adherent to the
surrounding tissues, the intima is composed of a basement membrane and endothelium. What vessel is it?
A *Nonmuscular vein
B Muscular vein with underdeveloped muscular elements
C Muscular artery
D Arteriole
E Artery of mixed type
During morphological investigation in a histological specimen one can see irregularly shaped vessel tunica media of which
formed by bundles of smooth muscle cells and layer of connective tissue. What type of vessel is it?
A *Large vein (muscular type)
B Muscular artery
C Lymphatic vessel
D Venule
E Arteriole
A specimen of the pia mater shows a vessel with no middle membrane in its wall, its outer membrane adheres to the
surrounding tissues, the inner membrane is made up of the basal membrane and endothelium. Specify this vessel:
A *Fibrous vein
B Muscular vein with weakly developed muscular elements
C Muscular artery
D Arteriole
E Mixed artery
In histological slide are present blind ended vessels which have shape of oblate tubes. They don’t have basal membrane and
pericytes. The outer surface of the endothelium is attached to the surrounding connective tissue by anchoring fibers. What type
of vessel is it?
A. *Lymph capillary
B. Blood capillary
C. Arterioles
D. Venules
E. Arterio-venous shunts
A histological specimen shows a blood vessel. Its inner coat is composed by endothelium and subendothelial layer. The middle
coat consists of smooth muscle cells bundles. Tunica adventitia well developed and consists of loose connective tissue and
some smooth muscle cells. Such morphological characteristics are typical for which vessel?
A *Muscular vein (large vein)
B Muscular artery
C Non-muscular vein
D Mixed artery (small artery)
E Elastic artery
Cardiovascular system. Heart.
During myocardium contraction in sarcoplasma of cardiac muscle cells concentration of calcium are increased. Which
structure participate in calcium storage
A. *L systems
B. Lysosomes
C. Ribosomes
D. T-systems
E. Nucleolus
Patient A. 40 years old had myocardial infarction of left ventricle. What morphological components of the heart wall will
replace this damage?
A. *Proliferation of connective tissue cells
B. Intracellular regeneration of contractive cardiac muscle cells
C. Proliferation of contractive cardiac muscle cells
D. Proliferation of conducting cells
E. Proliferation of contractive cardiac muscle cells and conducting cells
As a result of thrombosis of left coronary artery the group of contractive cardiac muscle cells have been destroyed. At the
expense of which cells reparation in the area of damage will occur?
A. *Fibroblasts
B. Cardiac muscle cells
C. Myosymplast
D. Myosatelitocytes
E. Smooth muscle cells
In a slide revealed organ of cardio-vascular system. One of it coats is build by fibers which formed anastomosis between each
other. They formed by cells which connected with the help of intercalated disks. Name this organ of cardio-vascular system.
A. *Heart
B. Large vein (muscle type)
C. Muscle artery
D. Elastic artery
E. Arteriole
In a histological specimen is represented heart wall. In one of its layers are found contractive, conducting and secretory muscle
cells, endomisium and blood vessels. Which layer of the heart is it?
A. *Myocardium of the atrium
B. Endocardium of the ventricles
C. Epicardium
D. Adventitia
E. Pericardium
Patient A. 40 years old had myocardial infarction of left ventricle. What morphological components of the heart wall will
replace this damage?
A. *Proliferation of connective tissue cells
B. Intracellular regeneration of contractive cardiac muscle cells
C. Proliferation of contractive cardiac muscle cells
D. Proliferation of conducting cells
E. Proliferation of contractive cardiac muscle cells and conducting cells
After myocardial infarction morphological structure of the heart wall was restored. At the expense of which tissue did
regeneration occur?
A. *Connective
B. Smooth muscle
C. Striated muscle
D. Epithelial
E. Nervous
Large cells with light cytoplasm and eccentrically located nucleus are revealed in the histological specimen of the heart wall
between endocardium and myocardium. What kind of heart cells have following morphologic signs?
A. *Purkinje cells
B. Pacemaker cells
C. Cardiac muscle cells
D. Endocrine cells
E. Adipose cells
In a heart specimen there are detected cells of squared shape, 80-120 micrometers in size, with a centrically positioned nucleus
and well-developed myofibrils connected with the help of intercalated disks. What function is connected with these cells?
A. *Heart contraction
B. Nerve impulses conduction
C. Endocrine
D. Protective
E. Regenerative
Hematopoietic and immune defense organs Central lymphatic organs.
During postembryonal hematopoiesis in the red bone marrow the cells of one of the cellular differons demonstrate a gradual
decrease in cytoplasm basophilia as well as an increase in oxyphilia, the nucleus is being forced out. Such morphological
changes are typical for the following haemopoiesis type:
A *Erythropoiesis
B Lymphopoiesis
C Neutrophil cytopoiesis
D Eosinophil cytopoiesis
E Basophil cytopoiesis
An electronic microphotograph shows a macrophagic cell with erythrocytes at different stages of differentiation located along
its processes. This is the cell of the following organ:
A *Red bone marrow
B Thymus
C Spleen
D Tonsil
E Lymph node
A 46 year old patient was admitted to the hematological department. It was found that he had disorder of granulocytopoesis
and thrombocytogenesis processes. In what organ does this pathological process take pace?
A *Red bone marrow
B Thymus
C Spleen
D Lymphatic ganglion
E Palatine tonsil
Medullary substance of a hemopoietic organ's lobule in a histological specimen is lighter coloured and contains epithelial
bodies. What organ are these morphological preperties typical for?
A *Thymus
B Lymph node
C Spleen
D Liver
E Kidney
Patient has disordered processes of erythropoiesis, granulocytopoiesis, monocytopoiesis and thrombocytopoiesis. Which
hematopoietic organ had been affected?
A. *Red bone marrow
B. Thymus
C. Spleen
D. Lymph node
E. Tonsil
During histological investigation of 40 years old man thymus was revealed decreasing of the thymus parenchyma, increasing
of adipose and loose connective tissue, enriching of Hassall’s corpuscles while weight of the organ stay the same. What is the
name of this phenomenon?
A. *Age involution of the thymus
B. Accidental involution of the thymus
C. Thymus hypotrophy
D. Thymus dystrophy
E. Thymus atrophy
In the slide of the human red bone marrow smear between myeloid row of cells and adipose cells one can find star shaped cells
with oxyphilic cytoplasm which contacts with each other by means of their processes. Call these cells.
A. *Reticular
B. Fibroblasts
C. Macrophages
D. Dendrite cells
E. Osteocytes
The child was born with immunodeficiency. Cell mediated immunity has been affected that has caused often viral infections.
Which organ has been damaged?
A. *Thymus
B.
C.
D.
E.
Red bone marrow
Lymph node
Spleen
Tonsil
A specimen of the human red bone marrow smear revealed accumulation of giant cells located near sinusoidal capillaries. Call
formed elements of blood which formed from these cells.
A. *Platelets
B. Red blood cells
C. White blood cells
D. Lymphocytes
E. Monocytes
Under condition of experiment in the body of investigated animal was injected antibody against thymus hormones. Which cells
differentiation will be affected first of all?
A. *T lymphocytes
B. Monocytes
C. Plasma cells
D. Macrophages
E. B lymphocytes
During infections and intoxication in the lobules of thymus amount of epithelioreticular cells and Hassall’s corpuscles are
increased and area of medulla became larger. Give name of these changes in the thymus.
A. *Accidental involution
B. B immunodeficiency
C. Thymico-lymphatic condition
D. Age involution
E. T immundeficiency
In the experiment in the human red bone marrow ribosomes in the polychromatophilic erythroblasts were destroyed. Which
specific protein synthesis will be disordered?
A. *Globin
B. Fibrinogen
C. Collagen
D. Elastan
E. Laminine
Slide of hematopoietic organ is under investigation. This organ consists of differently shaped lobules. In each of the lobule
presents cortex and medulla. Which organ posses these morphological characteristics?
A. *Thymus
B. Lymph node
C. Spleen
D. Tonsils
E. Appendix
In the electron microphotograph one can see macrophage along processes of which situated erythrocytes at different stages of
differentiation. What organ is it?
A. *Red bone marrow
B. Thymus
C. Spleen
D. Tonsil
E. Lymph node
In the slide one can see the organ stroma of which consists of reticular tissue, adipose cells, macrophages and osteogenic cells.
What organ is represented in the slide?
A. *Red bone marrow
B. Spleen
C. Thymus
D. Lymph node
E. Tonsil
In the specimen one can see an organ which consists from lobules and stroma includes epitheliocytes with processes. What
organ is represented in the slide?
A.
B.
C.
D.
E.
*Thymus
Red bone marrow
Spleen
Tonsil
Lymph node
A 46 year old patient was admitted to the hematological department. It was found that he had disorder of granulocytopoiesis
and thrombocytogenesis processes. In what organ does this pathological process take pace?
A *Red bone marrow
B Thymus
C Spleen
D Lymphatic ganglion
E Palatine tonsil
Medullary substance of a hemopoietic organ's lobule in a histological specimen is lighter colored and contains epithelial
bodies. What organ are these morphological properties typical for?
A *Thymus
B Lymph node
C Spleen
D Liver
E Kidney
Hematopoietic and immune defense organs Secondary (peripheral) lymphatic organs
In a histological specimen parenchyma of an organ is represented by lymphoid tissue that forms lymph nodes; the latter are
arranged in a diffuse manner and enclose a central artery. What anatomic formation has such morphological structure?
A *Spleen
B Tonsil
C Lymph node
D Thymus
E Red bone marrow
Examination of a patient who was exposed to the ionizing radiation revealed damage of white pulp. What cells of white pulp
undergo pathological changes?
A *Lymphocytes
B Neutrophilic leukocytes
C Basophilic leukocytes
D Monocytes
E Tissue basophils
A histological specimen presents an organ that has both cortical and medullary substance. Cortical substance consists of an
external zone that contains lymph nodules as well as of a paracortical zone. Medullary substance contains medullary cords,
sinuses and trabeculae. What organ possesses these morphological signs?
A *Lymph node
B Spleen
C Kidney
D Thymus
E Adrenal glands
In the slide which was made from the spleen one can see white and red pulp in the base of which is rest special tissue which
formed their stroma. What tissue is it?
A. *Reticular connective tissue
B. Dense connective tissue
C. Adipose tissue
D. Muscle tissue
E. Nervous tissue
In the histological cross section of the lymph node in the experimental animal after antigen stimulation in the medullary cords
one can find huge amount of cells with intensively basophilic cytoplasm, eccentrically positioned nucleus with chromatin
giving the illusion of the spokes of the wheel and light area of cytoplasm near it. Call these cells.
A. *Plasma cells
B. Macrophages
C. Fibroblasts
D. Adipose cells
E. Mast cells
In the biopsy sample of the lymph node were revealed focuses of increased formation of plasma cells. Antigen depended
stimulation of which immune cells have caused their formation?
A. *B lymphocytes
B. T lymphocytes
C. Macrophages
D. Dendrite cells
E. Interdigital cells
15 years old patient during tonsillitis has enlarged his tonsils. Which histological structures of these organs take place in
immune protection of the body as a response of streptococcus invasion?
A. *Lymphatic nodules
B. Stratified squamous keratinized epithelium
C. Stratified squamous non keratinized epithelium
D. Loose connective tissue
E. Crypt
In the specimen was revealed an organ in the reticular stroma of which situated blood formed elements and seen lymphoid
formation. What organ is this?
A. *Spleen
B.
C.
D.
E.
Lymph node
Tonsil
Thymus
Red bone marrow
Student got 2 histological specimens. They both have lymphatic nodules. First slide has only follicles but second one has
follicles with eccentrically positioned vessel. Determine these slides.
A. *First-lymph node, second-spleen
B. First-red bone marrow, second -spleen
C. First thymus, second- spleen
D. First liver, second -lymph node
E. First-liver, second -spleen
In a specimen was revealed roundish formation of the lymphocytes with a central artery in the center. What organ is it?
A *Spleen
B kidney
C Lymph node
D Thymus
E Red bone marrow
One has done histological section through lymph node. In the slide one can see enlargement of it paracortex. Proliferation of
what cells of lymph node have caused this process?
A. *T lymphocytes
B. Dendritic cells
C. Plasma cells
D. Macrophages
E. Reticular cells
In a microscopic specimen is a bean-shaped organ which has cortical and medullar substance. Cortical substance is represented
by separate spherical nodules 0,5-1 mm in diameter, medullar substance – by medullary cords. What organ is this?
A. *Lymph node
B. Kidney
C. Thymus
D. Adrenal gland
E. Spleen
In the specimen one can see an organ where lymphocytes formed 3 types of lymphoid structures such as lymphatic nodules,
medullary cords and sinuses. What organ is it?
A. *Lymph node
B. Spleen
C. Thymus
D. Tonsil
E. Red bone marrow
Morphological investigation of the spleen revealed activation of immune reactions in the organism. In which structures of this
organ do antigen depended proliferation of T lymphocytes begin?
A. *Periarterial sheath of white pulp
B. Central zone of white pulp
C. Germinal center
D. Marginal zone of white pulp
E. Red pulp
Hematopoietic and immune defense organs Intercellular communication in immune reactions
To prevent epidemic of California virus vaccine (heterogenous protein) was injected in the organism of human. What cells will
take place in specific immunity?
A. *Lymphocytes
B. Adipose cells
C. Pigmentocytes
D. Fibroblasts
E. Adventitial cells
A patient with clinical presentations of primary immunodeficiency displays disturbance of antigen-presenting function by
immunocompetent cells. What cells may have structure defect?
A *Macrophages, monocytes
B T-lymphocytes
C B-lymphocytes
D Fibroblasts
E 0-lymphocytes
Burn wound was covered with pig skin (heterotransplantation). Call effector cells, which will rejected transplant (pig skin).
A. *T killer
B. T helper
C. T suppressor
D. B lymphocytes
E. Natural killer
296. Plasma cell produces specific antibody for specific antigen. During injection of antigen quantity of plasma cell increased.
At the expense of what blood cells occurred increasing of plasma cells quantity?
A. *B lymphocytes
B. Eosinophils
C. Basophils
D. T lymphocytes
E. Monocytes
Rejection of transplant developed in the patient after transplantation of heterogenous kidney. Call main effector cells, which
take place in this immune reaction?
A. *T killer
B. B lymphocytes
C. T suppressor
D. T helper
E. Plasma cells
During heterotransplantation of the organ was revealed rejection of transplant. What blood cells will ensure this process?
A. *T killer
B. T helper
C. T suppressor
D. T 0 lymphocytes
E. T memory cells
Endocrine system. Hypothalamus and hypophysis
The aim of the morphological study was to investigate an endocrine gland with parenchyma consisting of epithelium and
neural tissue. In the epithelial trabeculae the study revealed two types of cells: chromophile and chromophobe. Identify this
organ:
A *Pituitary gland
B Adrenal gland
C Hypothalamus
D Thyroid gland
E Parathyroid gland
A 32-year-old patient consulted a doctor about the absence of lactation after parturition. Such disorder might be explained by
the deficit of the following hormone:
A *Prolactin
B Somatotropin
C Vasopressin
D Thyrocalcitonin
E Glucagon
Examination of a 32 year old patient revealed disproportional skeleton size, enlargement of superciliary arches, nose, lips,
tongue, jaw bones, feet. What gland's function was disturbed?
A *Hypophysis
B Epiphysis
C Pancreas
D Thyroid
E Suprarenal
Patient K., 35 years old complains about permanent thirst, bad appetite. He drinks every day 9 L of fluid. Daily diuresis
increased, urine discolored. Most probable reason of such pathology development in this patient is:
A. *Hypothalamic nucleuses
B. Epithelia of nephron tubules
C. Adenohypophysis
D. Pineal gland
E. Basal membrane of glomerulus capillaries
After sepsis 27 years old patient has bronze color of the skin which typical for Addison disease. Mechanism of hyper
pigmentation based in increasing of hormone secretion.
A. *Melanostimulating
B. Somatotropic
C. Gonadotropic
D. B lipotropic
E. thyreotropic
Patient has suffered from hypothyroidism for 7 years. Deficiency of thyrotropic hormones was revealed. What cells of
adenohypophysis will be changed?
A. *Thyrotropes
B. Gonadotropes
C. Corticotropes
D. Somatotropes
E. Mammotropes
On the background of deficiency of sex hormones in 30 years old female was revealed increased amount of follicle stimulating
hormone. What cells synthesized this hormone?
A. *Gonadotropes
B. Thyrotropes
C. Corticotropes
D. Somatotropes
E. Mammotropes
In the specimen of adenohypophysis between endocrine cells one can see cells cytoplasm of which is stained oxyphilly. These
cells secrete prolactin. Call these cells.
A. *Mammotropes
B. Thyrotropes
C. Adrenocorticotropes
D. Gonadotropes
E. Pituicytes
Cessation of bleeding after parturition connects with action of oxytocin to the uterus wall. Which layer of the organ does react
at the action of this hormone?
A. *Myometrium
B. Endometrium
C. Perimetrium
D. Parametrium
E. Submucous
For morphological investigation endocrine gland was represented. Parenchyma of this gland consists of epithelium and
nervous tissue. In epithelial trabeculae revealed two types of cells chromophiles and chromophobes. Name this organ.
A. *Hypophysis
B. Adrenal gland
C. Hypothalamus
D. Thyroid gland
E. Parathyroid gland
40 years old woman has weak labor activity caused by weak contraction of myometrium. What hormone should be injected to
help this woman?
A. *Oxytocin
B. Hydrocortisone
C. Dexametasone
D. Aldosterone
E. Prednisolone
Experimental animal produces big amount of urine and have strong thirst. Urine doesn’t have sugar. What cells disordered?
A. *Neurosecretory cells of supraoptic nucleus
B. Follicular endocrine cells of the thyroid gland
C. Principal cells
D. Endocrine cells of the zona glomerulosa of adrenal gland
E. Endocrine cells of the medullary region of adrenal gland
50 years old patient complains about enlargement of ears, nose and hands size. Hyperfunction of which gland does these
symptoms give?
A. *Hypophysis
B. Thyroid gland
C. Sex glands
D. Adrenal glands
E. Pineal gland
During X ray examination of the bones of the base of the cranium were revealed increasing of the cavity of sella turcica and
thinning of processus clinoideus anterior and damaging of different areas of sella turcica. Tumor of what endocrine gland can
cause such damaging of bones?
A. *Hypophysis
B. Thymus
C. Pineal gland
D. Thyroid gland
E. Adrenal gland
Endocrine system. Pineal gland. Adrenal gland.
Microscopic examination of a parenchymatous organ revealed that its epithelial cords formed glomerular, fascicular and
reticular zones. The central part of the organ was presented by accumulations of chromaffin cells. Specify this organ:
A *Adrenal gland
B Thyroid gland
C Epiphysis
D Liver
E Hypophysis
In a histological specimen of adrenal cortex there are petite polygonal cells that form roundish clusters and contain some lipid
inclusions. What part of adrenal is presented in this histological specimen?
A *Glomerulosa zone
B Intermedial zone
C Fasciculata zone
D Reticularis zone
E A patient has been given high doses of hydrocortisone for a long time. This caused atrophy of one of the adrenal cortex zones.
Which zone is it?
A *Fasciculata
B Glomerulosa
C Reticularis
D Glomerulosa and reticularis
E In the slide endocrine system organ is represented. It surrounded by connective tissue capsule which extends trabeculae in the
center of the organ and formed lobules. Each lobule contains two types of cells Neurosecretory pinealocytes – polygonal cells
with processes located in the center and glial cells (astrocytes) located in the periphery. What organ is this?
A. *Pineal gland
B. Hypophysis
C. Hypothalamus
D. Thyroid gland
E. Medullary region of the adrenal gland
Characterizing stress students made an inaccuracy by telling that synthesis of cortical region of adrenal gland glucocorticoids is
stimulated by hypophysis hormones. What clarification should he make?
A. *Adrenocorticotropic hormone
B. Somatotropin
C. Gonadotropic hormone
D. Mammotropic hormone
E. Thyrotropic hormone
In the specimen parenchymal organ is represented. External layer of the cortex of it is formed by glomeruli created by
endocrine cells. What organ is it?
A. *Adrenal gland
B. Lymph node
C. Spleen
D. Thyroid gland
E. Ovary
It is known that aldosterone regulates amount of sodium in the body. What cells of the adrenal gland do synthesize this
hormone?
A. *Cells of zona glomerulosa
B. Chromaffin cells producing epinephrine
C. Cells of zona reticularis
D. Cells of zona fasciculata
E. Chromaffin cells producing nor epinephrine
Endocrine system. Thyroid gland. Parathyroid gland
Parodontitis is treated with calcium preparations and a hormone that stimulates tooth mineralization and inhibits tissue
resorption. What hormone is it?
A *Calcitonin
B Parathormone
C Adrenalin
D Aldosterone
E Thyroxine
A child has disturbed enamel and dentine formation as a result of decreased content of calcium ions in his blood. What
hormone deficiency may cause such changes?
A *Thyreocalcitonin
B Somatotropin
C Thyroxin
D Parathormone
E Triiodothyronine
A patient has the sudden decrease of Са2+ content in blood. What hormone secretion will increase?
A *Parathormone
B Thyrocalcitonin
C Aldosterone
D Vasopressin
E Somatotropin
A child has abnormal formation of tooth enamel and dentin as a result of low concentration of calcium ions in blood. Such
abnormalities might be caused by deficiency of the following hormone:
A *Parathormone
B Thyrocalcitonin
C Thyroxin
D Somatotropic hormone
E Triiodothyronine
Microscopic study of an endocrine gland revealed that its parenchyma consisted of follicular structures. Their wall was formed
by monolayer cubic epithelium, and their cavity was filled up with oxyphilic substance. What hormone is secreted by this
gland?
A *Thyroxin
B Aldosterone
C Cortisol
D Parathormone
E Oxytocin
A 9 y.o. boy was admitted to the endocrinological department. This boy has already had several fractures of wrist extremities
due to bone brittleness. The function of the following endocrinal glands (gland) is disturbed:
A *Parathyroid
B Thyroid
C Thymus
D Adrenal
E Epiphysis
Clinical examination of a female patient revealed reduction of basal metabolism by 40%, gain in body mass, drop of body
temperature, face puffiness, sexual dysfunctions, inertness and apathy, lowered intelligence. These symptoms are caused by
dysfunction of the following endocrine gland:
A *Hypofunction of thyroid gland
B Hypofunction of parathyroid glands
C Hypophysis hyperfunction
D Epiphysis hypofunction
E Hyperfunction of thyroid gland
After a surgical procedure an experimental animal died from intense convulsions. What endocrinal glands were extracted?
A *Parathyroid
B Thyroid
C Adrenal
D Ovaries
E Testicles
Examination of a patient who complains about deglutitive problem revealed a tumor-like eminence 1-2 cm in diameter on the
tongue root in the region of the cecal foramen. These are overgrown remnants of the following gland:
A *Thyroid
B Parathyroid
C Adenohypophysis
D Thymus
E Sublingual
Parodontitis is treated with calcium preparations and a hormone that stimulates tooth mineralization and inhibits tissue
resorption. What hormone is it?
A *Calcitonin
B Parathormone
C Adrenalin
D Aldosterone
E Thyroxin
In the endocrine gland specimen one can see roundish structures varying sizes wall of which is formed with one layer of
epithelial cells lying on the basal membrane. These structures contain homogeneous non cellular mass in the middle of them.
What gland is it?
A. *Thyroid gland
B. Adrenal gland, cortex
C. Parathyroid gland
D. Anterior part of hypophysis
E. Posterior part of hypophysis
During inspection of patient oral cavity dentist noticed significant tremor of the tongue. Except it exophthalmos was present.
Doctor suggested him to contact endocrinologist. During examination patient was diagnosed with Basedow’s disease.
Hyperfunction of what cells caused this problem?
A. *Follicular cells
B. Parafollicular cells
C. Principal cells
D. Endocrine cells of adrenal gland zona glomerulosa
E. Endocrine cells of adrenal gland zona fasciculata
During operation by mistake in the patient endocrine gland was removed. It caused decreasing of the calcium blood level.
What gland was removed?
A. *Parathyroid
B. Hypophysis
C. Adrenal
D. Thyroid
E. Pineal
During operation at thyroid gland two from four parathyroid glands were mistakenly removed. It caused decreasing of the
calcium blood level. What cells are target for parathyroid gland hormone which increase level of calcium?
A. *Osteoclasts
B. Osteocytes
C. Osteoblasts
D. Fibroblasts
E. Chondrocytes
Two years old child has convulsions as a result of decreasing concentrations of calcium ions in the plasma. This is caused by
declining function of:
A. *Parathyroid glands
B. Hypophysis
C. Adrenal cortex
D. Pineal gland
E. Thymus
Young woman comes to an endocrinologist with complains about sleepiness, depression, fatigue, bad appetite and increasing
of the body weight. Disorder of what endocrine gland can cause such problems?
A.
B.
C.
D.
E.
*Thyroid
Pancreas
Adrenal cortex
Adrenal medulla
Ovary
Under action of harmful factors of environment in follicular cells lysosomes formation disordered. What part of hormone
production in the thyroid gland will be disordered?
A. *Proteolysis of phagocytized colloid from follicles
B. Colloid synthesis
C. Iodination of colloid
D. Resorption of colloid
E. Thyroglobulin synthesis
40 years old patient came to a doctor with complains about tachycardia, exophthalmos, fatiguability, reduced weight of the
body. Increasing of what cells functions does this condition connect?
A. *Follicular cells
B. Parafollicular cells
C. Parathyroid cells
D. Apud cells
E. Acidophil endocrine cells
30 years old patient was diagnosed with thyroid gland hyperfunction. What shape do follicular cells have in the follicles?
A. *Columnar
B. Polygonal
C. Squamous
D. Spindle
E. Cuboidal
42 years old patient after resection of thyroid gland had convulsions. After injection of calcium preparation she got relief.
Disorder of what endocrine glands have caused this condition?
A. *Parathyroid
B. Adrenal
C. Ovary
D. Hypophysis
E. Pineal
Digestive system. Mouth. Tongue. Tonsils.
There is a specimen of soft palate where both oral and nasal surfaces can be seen. It was revealed that oral cavity had damaged
epithelium. What epithelium is damaged?
A *Multistratal squamous nonkeratinizing
B Multistratal cubical nonkeratinizing
C Multistratal prismatic nonkeratinizing
D Multistratal squamous keratinizing
E Multirowed ciliated epithelium
A child damaged the lateral surface of his tongue. What lingual papillas are most likely to be damged?
A *Foliate
B Conic
C Vallate
D Filiform
E Fungiform
A histological specimen of an oral cavity organ demonstrates that the organ's anterior surface is lined with multilayer
squamous nonkeratinous epithelium, and its posterior surface - with multiserial ciliated epithelium. What organ is it?
A *Soft palate
B Gingiva
C Hard palate
D Lip
E Cheek
One of the structure of oral cavity has a few folds of mucous membrane in the lamina propria with numerous lymphoid
follicles. What kind of organ is it?
A *Tonsils
B Tongue
C parotid gland
D Sublingual gland
E submandibular gland
The patient initiated a sense of taste. The overall sensitivity persists. Which papillae of the tongue are not damaged?
A * Filiform
B Circumvallate
C Fungiform
D Foliate
E All
A 53 years old patient complaining with the deterioration of taste sensitivity. During the examination doctor noticed the
phenomenon of mucosal atrophy of certain areas of the mouth. Where are the most likely observed morphological changes?
A * On the upper surface of the tongue
B On the lower surface of the tongue
C At the root of the tongue
D On the hard palate
E On the gums
During an injury one of the areas of the mouth was damaged, it has maxillary, intermediate and mandibular area. Which organ
was damaged?
A * Cheek
B Tongue
C Lips
D Hard palate
E Soft palate
In the biopsy of oral mucosa revealed morphological signs of the gums. What are the structural features of the mucous
membrane of the gums can be observed in normal?
A * Quiescent adherent to the periosteum, lamina propria forming high papillae, absent of muscular plate
B Chubby adherent to the periosteum, well defined muscular plate
C No muscular plate, submucosa is well-developed
D No muscular plate and lamina propria
E Contains many small salivary glands
In histological slide were identify the part of the oral cavity, which is based on bone tissue. It is covered by mucous membrane,
which shows stratified squamous epithelium. In the formation distinguish fat, glandular and marginal zone. In all areas of the
lamina propria of mucosa collagen fibers form a powerful beam that intertwines in the periosteum. What kind of structure is
represented in the sample?
A * Hard palate
B Gums
C Lips
D Cheek
E Tongue
In newborn baby was found median cleft lip and upper jaw. Abnormalities of which processes is caused this defect?
A * Nonunion of the medial nasal processes.
B Nonunion of the medial nasal processes of the maxilla.
C Nonunion of the lateral nasal processes of the maxilla.
D Cleft palatine processes.
E Cleft maxillary processes.
During the study of histological slides of oral mucosa was found that the stratified squamous epithelium infiltrated by
lymphocytes. What is the area of the mouth is most likely represented in the sample?
A * Mucous membrane of the tonsils
B Mucous membrane of the lips
C Mucous membrane of the cheek
D Mucous membrane of the hard palate
E Mucous membrane of the gums
351. In the study of histological preparations of the mouth shows that it has in its structure three parts: skin, mucous and
intermediate parts. Basis of structure forms striated muscle. What is the structure of oral cavity?
A * Lips
B Hard palate
C Soft palate
D Tongue
E Gums
Dentist during examination of the oral cavity of the patient noticed that his tongue is covered with a whitish bloom. What
histological structures involved in its formation?
A * Epithelium of filiform papillae
B Epithelium of foliate papillae
C Epithelium of circumvallate papillae
D Epithelium of fungiform papillae
E Lingual tonsils
The baby receives breast milk. What are the histological structure of oral cavity adapted for breast nipple stimulation causing
reflex of milk?
A * Epithelial villi of the lips
B Keratinizing stratified squamous epithelium of the lips
C Connective tissue papillae of the lips
D Mushroom-shaped papillae of the tongue
E Foliate papillae of the tongue
In histological slides was defined structure of the oral cavity, represented by mucous membrane, which is free of attached and
which is firmly adherent to the periosteum. Epithelium - stratified squamous keratinized. Lamina propria form long papillae
deeply resort in the epithelium. What kind of structure is it?
A *Gums
B Hard palate
C Lip
D Cheek
E Tongue
In the newborn were found a defect in the sagittal line of the soft palate. Which process is displayed?
A * Swallowing
B Chewing
C Digestion
D Breathing
E articulation
In histological slide identified crypts of the tonsils, the epithelium is infiltrated by leukocytes. What kind of the epithelium is a
part of this structure?
A *Stratified squamous nonkeratinized.
B Simple columnar.
C Stratified cuboidal.
D Stratified squamous keratinized.
E Ciliated.
The patient, 40 years old, suffering from heart attacks. A doctor appointed him receiving nitroglycerine under the tongue. What
are the structural features of the mucous membrane of the mouth primarily determine the opportunity of medication?
A *Permeability of the stratified squamous nonkeratinized epithelium
B Permeability of the stratified squamous keratinized epithelium
C Permeability of the stratified squamous epithelium
D The presence of papillae of the tongue
E The presence of salivary glands
The patient, 30 years old, came to the doctor with complaints of fever up to thirty eight degrees, weakness, pain in the throat.
The examination revealed that the patient's tongue is covered with white bloom. What are the histological structure of the
tongue involved in the formation of plaque?
A * Filiform papillae
B Foliate papillae
C Fungiform papillae
D Circumvalate papillae
E Connective tissue papillae of the tongue
In the histological slide the head end of the embryo 5 weeks of determined gill arch. Indicate what develops from the first pair
of data structures?
A * Mandibular and maxillary processes.
B Mandibular processes.
C Maxillary processes.
D The external auditory canal.
E Thyroid cartilage.
Excalation of which parts of the facial skull in the embryonic period leads to malformations such as "cleft palate"?
A * Palatine processes
B Frontal processes
C Frontal and maxillary processes
D Mandibular processes
E Mandibular and palatine processes
Digestive system. Salivary glands.
A histological specimen of a mandibular gland shows an exctretory duct. Mucous membrane of the duct is lined with cubic
epithelium whose cells have weakly developed organellas. What excretory duct is it?
A *Intercalated
B Striated
C Interlobular
D Common excretory
E A microspecimen of parotid gland presents secretory acines with serous cells that synthesize mostly enzymes. According to the
chemical composition classification, the parotid gland relates to the following glands:
A *Serous
B Mucous
C Seromucous
D Enzymatic
E Examination of a microspecimen made of an unknown organ revealed some acini that contained 10-15 cone cells with
basophilic cytoplasm, round nucleus and well developed granular endoplasmic reticulum. An acinus is surrounded by a basal
membrane with myoepithelial cells localized in its splitting. What organ is the slice made of?
A *Parotid gland
B Pancreas
C Lungs
D Sublingual gland
E Liver
A microspecimen of the submandibular salivary gland shows some basket-shaped cells concentrated around the acines and
excretory ducts. These cells surround bases of the serous cells and are called myoepitheliocytes. These cells relate to the
following tissue:
A *Muscular tissue
B Epithelial tissue
C Neural tissue
D Special connective tissue
E Loose fibrous connective tissue
What substance makes the saliva viscous and mucous and performs protective function, including protection from mechanical
injury of mouth mucous membrane?
A *Mucin
B Glucose
C Kallikrein
D Amylase
E Lysozyme
In course of embryogenesis maxillary and mandibular processes grew together with a delay. What development anomalies
should be expected in this case?
A *Macrostomia
B Microstomia
C Cleft palate
D Gothic palate
E Cleft of superior lip
It is known that the submandibular salivary gland has a slimy end sections, consisting of mucocytes. What are the signs
characteristic of these cells?
A * Flattened nucleus and light cytoplasm
B Basophilic cytoplasm
C Rounded nucleus in the center of the cell
D Microvilli
E Basal striation
Some diseases of the salivary glands caused by disruption of their excretory ducts. Which types are distinguished in the
excretory ducts of salivary glands?
A * Intra-, interlobular duct and common duct
B Intralobular ducts, striated and common duct
C Glistering, striated and common duct
D Intra-, interlobular ducts
E Intralobular ducts and outside the gland ducts
In histological slide of the gland determined only serous end sections. In the interlobular connective tissue visible ducts, lined
with double-layer or multilayered epithelium. Determine this structure.
A * Parotid gland
B Submandibular salivary gland
C Pancreas
D Sublingual salivary gland
E Liver
Digestive system. Teeth.
In histological slide of mandibular tooth germ of embryo turns in which the dental papilla formed by small colored basophilic
stellate cells. What tissue forms part of these dental germs?
A * Mesenchyme
B. Epithelial
C Reticular
D Cartilage
E Bone
On electronic microscopy of periodontal tissue fibers were found that one of their ends dipped in cement tooth root, and others
- in the periosteum of the alveolar process. What kind of fibers they are?
A * Sharpey fibers
B Korff fibers
C Ebner fibers
D Purkinje fibers
E Argyrophilic fibers
On the histological slide were observed around the tooth periodontal thick bundles of collagen fibers that provide fixing the
tooth in the dental alveoli. What are these fibers in the cervical area of the tooth?
A * Circular
B Oblique
C Apical
D Elastic
E Myelinated
On the histological slide of periodont were observed dense connective tissue composed of thick bundles of collagen fibers and
provides a fastening tooth in the dental alveoli. This structurec has a special name:
A * Dental fiber connections
B Breakthrough fiber
C Myelinated fibers
D Korff fibers
E Ebner fibers
42 years old patient, addressed to the dentist complaining of severe toothache. After examining the patient the doctor found
inflammation of the dental pulp. What tissue forms the pulp of the tooth?
A * Loose connective tissue
B Amorphous dense fibrous connective tissue
C Dense fibrous connective tissue framed
D Reticular connective tissue
E Mesenchyme
To keep the tooth in the hole in the periodontal alveolar processes are bundles of collagen fibers that are arranged in different
directions. How periodontal fibers oriented on the sides of the root?
A * Obliquely
B Vertical
C Horizontal
D Perpendicular
E Circularly
At the main part of the crown, neck and root of the tooth are present a dentin, whose length may increase with age, possibly as
part of his recovery from injury. What structures provide these processes?
A * Odontoblasts
B Dentinal tubules
C Perytubular dentin
D Ameloblasts
E Cementoblasts
In the molars is visible a tissue, located at the upper part of the roots and in places of their branching. The tissue contains cells
lying in lacunae and numerous collagen fibers with radial or longitudinal direction. Name this tissue.
A * Cell cement
B Reticular fibrous bone tissue
C Dentin
D Enamel
E Dense connective tissue
In the enamel on the edge of the dentin found non calcified areas that are often a place of infection in the tooth. How to call
such structures?
A * Enamel bundles
B Enamel prisms
C Enameloblasts
D Dentynoblasts
E Toms Fibers
In histological slide, showing the histogenesis of tooth visible deposition of hydroxyapatite crystals in the form of globules.
For what tooth characteristic this type of mineralization?
A * Dentin
B Enamels
C Periodontal
D Cement
E Pulp
The child complains of a toothache. Dentist stated carious enamel damage. Number of minerals which decreases in caries
damage:
A * Phosphorous, fluorine and calcium;
B Sodium, calcium, potassium,
C Potassium, phosphorus, fluorine;
D Magnesium fluoride, calcium
E Phosphorus, magnesium, potassium
On histological slide grinding tooth enamel defined light and dark bands width 100 mm, directed radially. Identify the
formation of enamel.
A *Bands of Gunther Shreher
B Lines of Retius
C Perykimatiy
D Enamel prisms
E Enamel tufts
In histological slides were found acellular cement. In which part of the tooth localized this tissue?
A *On the lateral surface of the tooth root
B On the surface of the crown
C Forms a layer of coronal pulp
D At the top of the tooth root
E In the pulp canal
During the examination of the patient was found abnormalities of enamel. With damage of what structural components of tooth
germ is it occurs?
A * Inner epithelium of enamel organ
B Intermediate layer of the enamel organ
C Pulp of enamel organ
D Outer epithelium of enamel organ
E Neck of enamel organ
During the development of the primary tooth dentin tissue is laid. What is the source of its development?
A * Dental papilla
B Dental pouch
C Tooth plate
D Inner cells of the enamel organ
E Outer cells of the enamel organ
In the study of histological slides dental pulp was observed in the connective tissue dominated by bundles of collagen fibers
odontoblasts layer thin intermediate layer is weak. In which area of the tooth pulp has the following features?
A * Root pulp
B Coronal pulp
C Layer Weil
D Peripheral layer of pulp
E The central layer of pulp
In the investigation of the chemical composition of dentin was found that in some areas it contains high content of minerals.
Any damage to tooth decay much faster it breaks down, leading to the extension tubes and increasing the permeability of
dentin. What type of dentin is it?
A * Peritubular dentin
B Interglodular dentin
C Predentine
D Mantle dentin
E Circumpulpal dentin
On histological slide grinding crown dentine in the intercellular substance is determined by a small number of collagen fibers
(fibers of Korf) coming in the radial direction. Name this layer of dentin.
A * Mantle dentine
B Circumpulpal dentin.
C Granular layer.
D Interglobular dentin.
E Predentine.
In histological preparation grinding teeth is determined by cell-free tissue, consisting of intercellular substance pierced tubes,
which are located in the cell processes. What structure is represented in the sample?
A * Dentine.
B Enamel.
C Pulp.
D Cement.
E Dense connective tissue.
Electron photomicrographs of transverse thin sections of tooth enamel lesions appear oval, polygonal or arched form,
consisting of compacted and organized hydroxyapatite crystals. What is this formation
A * Enamel prisms
B Lines of Retius
C Bands of Gunther - Shreher
D Perykimatium.
E Collagen fibers.
In histological slide of tooth germ enamel organ outer surface is rough, the cells of the inner layer of cells occurred polarity
(inversion nuclei). Determine the beginning of the process which preceded these changes.
A * Enamelogenesis
B Dentinogenesis
C Pulpogenesis
D Cementogenesis
E Periodontal development
On histological slide of the tooth in one of the tissues in the intercellular substance visible collagen fibers with radial and
tangential direction. Decide for which tissue histogenesis is this a typical picture?
A * Dentin
B Enamel
C Cementum
D Pulp.
E Dense connective tissue.
On histological slide of the tooth is determined enamel organ in the form of "bell", which shows the external cube-shaped
enamel cells, high internal prismatic cells and centrally located cells with processes that form a network. What is the period of
tooth represented in the sample?
A * During the formation and differentiation of dental germs.
B Period bookmarks dental germs.
C During the formation of the tissues of the tooth crown.
D During the formation of the tissues of the tooth root.
E Odontiasis period.
In longitudinal grinding tooth dentin tubules seen. What is inside the tubules?
A * Processes of dentynoblasts.
B Processes of enameloblasts.
C Body of dentynoblasts.
D Fbroblasts.
E Elastic fibers
Before teeth come out first on their roots appears a solid tissue that looks like membrane reticulated bone. What tissue is it?
A *Cement
B Dentin
C Enamel
D Loose fibrous connective tissue
E Dense fibrous connective tissue
A histological specimen presenting a tooth slice shows that the intercellular dentin substance contains collagen fibers being
tangential to the dentinoenamel junction and perpendicular to the dentinal tubules (Ebner's fibers). This dentin layer is called:
A *Parapulpar dentin
B Mantle dentin
C Granular layer
D Interglobular dentin
E Secondary dentin
Histological examination of trasverse enamel slice revealed linear banding in form of concentric circles that is pointing at an
angle to the dentinoenamel junction. Name these structures:
A *Retsius' lines
B Hunter-Schreger's lines
C Enamel plates
D Enamel fascicles
E Enamel spindles
During the tooth development the enamel organ has prismatic cells with hexagonal intersection; the nucleus is situated in the
central part of the cell. What cells are meant?
A *Preenameloblasts
B Exterior enameloblasts
C Cambial cells
D Enamel pulp cells
E Preodontoblasts
During the embryogenesis of oral cavity the development of dental enamel was disturbed. What source of dental development
was damaged?
A *Epithelium
B Mesenchyma
C Mesoderma
D Dental saccule
E Dental papilla
Examination of a tooth slice of a 42 y.o. man revealed on the dentinal-enamel border some solid linear fusiform structures as
long as 1/3 of enamel depth. What structures were revealed?
A *Enamel spindles
B Denticles
C Enamel fascicles
D "Dead" tracts
E Carious damage
A histological specimen of mandible of an embryo shows a tooth germ with the dental papilla made up of small stellate
basophilic cells. What tissue forms this part of the tooth germ?
A *Mesenchyme
B Epithelial
C Reticular
D Cartilaginous
E Osseous
Examination of a 42-year-old patient suffering from paradontosis revealed some roundish calcified formations 2-3 mm in
diameter in the coronal pulp. Name these structures:
A *Denticles
B Interglobular dentin
C Interglobular spaces
D Sclerotic dentin
E Dead dentin
A histological specimen presents a developed tooth that has a coating resistant to acids, but it can be found only on the lateral
surfaces of the tooth. What coating is meant?
A *Cuticle
B Dentine
C Enamel pellicle
D Enamel
E Cement
Histological study of an extirpated pulp revealed some cylindrical cells in its peripheral layer. What are these cells called?
A *Odontoblasts
B Fibroblasts
C Monocytes
D Ameloblasts
E Myofibroblasts
In the process of tooth tissue histogenesis dentin wasn't formed in time for some reasons. What process of further histogenesis
will be delayed or will not take place at all?
A *Enamel formation
B Pulp formation
C Predentinal space formation
D Cellular cement formation
E Acellular cement formation
In animal experiments the dental germs destroyed the inner layer of the epithelium of the enamel organ. Development of which
tooth is broken?
A * Enamel
B Dentine
C Cementum
D Pulp
E Periodontal ligament
Digestive system. Pharynx. Esophagus.
A patient underwent gastroscopy that revealed insufficient amount of mucus covering the mucous membrane. This
phenomenon is caused by the dysfunction of the following cells of stomach wall:
A *Cells of prysmatic glandular epithelium
B Parietal cells of gastric glands
C Principal exocrinocytes of gastric glands
D Cervical cells of gastric glands
E Endocrinocytes
The 60 years old patient is suffering from chronic gastritis. At the endoscopy of the stomach observed changes in the
epithelium of the mucosa. Which type of epithelium covers of the stomach mucosa?
A * Simple columnar mucous epithelium
B Pseudostratified ciliated columnar epithelium
C Smple squamous epithelium
D Simple cuboidal epithelium
E Simple columnar brush-border epithelium
The patients with burnings of the esophagus was examined by the doctor and found that lesions of the mucous membrane are
not deep. Due to which layer of cells will take place regeneration of damaged epithelium.
A * Basal
B Spinous
C Grained
D Intermediate
E Surface
On histological slide represented a cross section of a hollow organ wall, which has a mucous membrane consist of
nonkeratinized stratified epithelium. What kind of structure is it?
A * Esophagus
B Duodenum
C Colon
D Uterus
E Appendix
On microscopic examination were presented striated muscle tissue of the digestive system. In what kind of organ the biopsy
was taken?
A * Esophagus
B Stomach
C Duodenum
D Ileum
E Appendix
Digestive system. Stomach.
On the electronic microphotographs of fundic glands were determined a large, pale and round to pyramidal cells. They have
one or 2 central nuclei and acidophilic cytoplasm. The many mitochondria indicate that their secretory activity is energydependent. What kind of cell they are?
A * Parietal (oxyntic) cells
B Mucous neck cells
C Undifferentiated cells
D Enteroendocrine cells
E Chief cells
Insulin injection did for assess the completeness of vagotomy accompanied by a significant increase of ph of gastric juice.
Which cells of gastric glands controls this process?
A * Parietal cells
B Enteroendocrine cells
C Chief cells
D Mucous neck cells
E Undifferentiated cells
On the histological sections of fundic glands we can see as large cells with acidophilic cytoplasm. What kind of component of
gastric juice is produced as a result of the activity of these cells?
A * Hydrochloric acid
B Pepsinogen
C Mucus
D Serotonin
E Gastrin
Under the action of harmful factors occurred local damage of the epithelium of the stomach. What kind of cell can provide
regeneration process?
A * Mucous neck cells
B Parietal cells
C Chief cells
D Enteroendocrine cells
E Mucocytes
The 20 years old patient suffers from rheumatism appointed of prolonged use of aspirin. What is the structural component of
the mucous membrane of the stomach is best suited to provide her protection from damage?
A * Simple columnar mucous epithelium
B Connective tissue
C Muscle tissue
D Multi-ciliated epithelium
E Stratified squamous nonkeratinized epithelium
The patient biopsy from the stomach shows histologically revealed a significant reduction or complete absence of parietal cells
in the glands. Which layer of mucous membrane they studied?
A * Pylorus
B Fundus of stomach
C Cardiac part
D Body of stomach
E–
In 42 years old patient after radiotherapy of cancer of the stomach developed pernicious anemia due to damage cells by
producing intrinsic factor. Which cells of fundic glands were damaged?
A * Parietal cells
B Mucous neck cells
C Surface mucous cells
D Enteroendocrine cells
E Chief cells
In the embryonic material was damaged endoderm. What kind of changes of development may arise in this process?
A * Stomach
B Heart
C Kidneys
D Aorta
E Salivary glands
Damaged by exposure to various factors of gastric mucosa can restore its integrity. By which cells of gastric glands can occur
their regeneration?
A * Mucous neck cells
B Parietal cells
C Paneth cells
D Chief cells
E Enteroendocrine cells
By ingestion of a foreign body in the stomach was damaged epithelium of the child. How the cell regeneration process
possible?
A * Mucous neck cells
B Chief cells
C Parietal cells
D Connective tissue cells
E Fat cells
During the fibrogastroscopy of the patient revealed damage of the mucosa layer. By which cells will occur in the treatment of
epithelial regeneration?
A * Poorly differentiated mucous neck cells
B Chief cells
C Parietal cells
D Enteroendocrine cells
E Paneth cells
During inflammatory diseases of the stomach damaged surface epithelium of the gastric mucosa. What kind of epithelium was
damaged?
A * Simple columnar mucous epithelium
B Stratified squamous nonkeratinized epithelium
C Stratified squamous keratinized epithelium
D Simple cuboidal epithelium
E Stratified cuboidal epithelium
Analysis of biopsy material of human gastric mucosa, gastritis patient showed a dramatic decrease in the number of parietal
cells. How will change the components of gastric juice?
A * Reduce acidity
B Increased acidity
C Increased gastric juice
D Reduction of gastric juice
E Reducing mucus production
Examination of a patient, suffering from atrophic gastritis, revealed megaloblastic anemia. The anemia is likely to be caused by
the deficiency of the following substance:
A *Gastromucoproteid
B Vitamin B6
C Vitamin B1
D Iron
E Erythropoietins
A patient underwent gastroscopy that revealed insufficient amount of mucus covering the mucous membrane. This
phenomenon is caused by the dysfunction of the following cells of stomach wall:
A *Cells of prysmatic glandular epithelium
B Parietal cells of gastric glands
C Principal exocrinocytes of gastric glands
D Cervical cells of gastric glands
E Endocrinocytes
During the examination of the patient's oral cavity dentist noticed that his tongue is rough hypertrophic nipples, deep furrows.
The doctor advised the patient to consult a gastroenterologist. The examination revealed that he had considerably increased
acidity of gastric juice. Hyperfunction of which cells in the glands of the mucous membrane of the stomach mainly caused this
condition?
A * Parietal cells of fundic glands
B Chief cells
C Additional mucocytes
D Goblet cells
E Exocrine pancreatocytes
The 60 years old patient is suffering from chronic gastritis. During an endoscopy of the stomach observed changes in the
epithelium of the mucosa. What epithelium has undergone a change?
A * Simple columnar mucous epithelium
B Connective tissue
C Muscle tissue
D Multi-ciliated epithelium
E Stratified squamous nonkeratinized epithelium
Digestive system. Small and large intestine.
In some diseases of the colon change correlation of number between epithelial cells of the mucosa. What types of cells
predominate in the epithelium of the crypts of the colon normally?
A * Goblet cells
B Columnar villous epithelial cells
C Endocrinocytes
D Cells with acidophilic granules
E Undifferentiated cells
Some diseases of the small intestine associated with dysfunction of exocrynocytes with acidophilic granules (Paneth cells).
Where are these cells located?
A * At the bottom of intestinal crypts
B On the apical side of intestinal villi
C On the sides of the intestinal villi
D In place of transition in villus crypt
E In the upper part of the intestinal crypts
During endoscopic examination of the patients with chronic enterocolitis (inflammation of the colon), there is no specific
structures reliefs of the small intestine. What components determine the relief features of the mucous membrane of this organ?
A * Circular folds, villi and crypts
B Felds, folds, holes
C Haustry, villi, crypt
D Oblique folds
E Villi
During the examination of the patients with diseases of the small intestine revealed disruption of the wall and membrane
digestion. With dysfunction of what kind of cells it is connecting?
A * Column with border
B Column without border
C Goblet
D Paneth cells
E Endocrinocytes
During the diseases of the mucosa layer of the small intestine suffers absorption function. What kind of epithelium is
responsible for this function?
A * Simple columnar epithelium
B Simple cuboidal
C Simple columnar
D Stratified squamous epithelium
E Stratified cuboidal
The student had a histological slide of the small intestine submucosa in which the foundation has a large number of glands.
Which part of the small intestine is it?
A * Duodenum
B Jejunum
C Ileum
D Ascending colon
E Descending colon
On the electronic microphotogram of crypts of the small intestine can be identified some cells. They lying in the bases of the
crypts, have many large acidofilic secretory granules. What is the name of these cells?
A * Paneth cells
B Enterocytes
C Goblet cells
D Undifferentiated cells
E Enteroendocrine cells
On a histological specimen of the small intestine in the lamina propria of mucosa revealed clumps of cells spherical shape with
large basophilic nuclei surrounded by a narrow rim of cytoplasm. In most of the central part of the light clusters and contains
fewer cells than peripheral. Which is the morphological structure of such clusters?
A * Lymphatic nodule
B Nerve bundle
C Fat cells
D Blood vessels
E Lymphatic vessels
On a histological specimen of submucosa of the small intestine is filled of endings of the protein secretory glands. Where was
the section of intestine done which is presented in the sample?
A * Duodenum
B Jejunum
C Cecum
D Ileum
E Appendix
In the cytoplasm of epithelial cells of the colon’s crypts were found dypeptydase and lysozyme. What kind of cells can produce
these enzymes?
A * Paneth cells
B Columnar epithelial cells
C Goblet cells
D A-cells
E S-cells
On a histological specimen of a wall of the digestive system in the lamina propria of the mucosa and submucosa were found
numerous lymphoid nodules. Name the structure.
A *Appendix
B Stomach
C Duodenum
D Ileum
E Colon
During a biopsy investigation of the wall of the small intestine has been taken out the part of mucosa layer. What epithelium
covers the mucosal surface of the organ?
A * Simple columnar epithelium with goblet cells
B Stratified squamous nonkeratinized epithelium
C Stratified squamous keratinized epithelium
D Simple cuboidal epithelium
E Stratified cuboidal epithelium
Digestive system. Liver.
An electron micrograph of internally lobed liver sinusoid represented cells, which can be seen in the cytoplasm granules with a
seal like fruit pit. We know that this is a natural killer cell. Which cell is represented.
A * Pit-cell
B Hepatocytes
C Endothelial cells of sinusoid
D Kupfer cells
E Fat-storing cell
In the patient from therapeutic department with severe liver pathology revealed violations of coagulation. What is the function
of the liver can be affected in this case?
A * Protein Synthesis
B Detoxication
C Endocrine
D Protective
E Bile formation
During proliferation of connective tissue in parenchyma of the liver (fibrosis) due to chronic diseases there is disruption of
blood circulation in classic liver lobule. What is the direction of blood flow in these structures?
A * From the periphery to the center
B From the center to the periphery
C Around lobule
D From the top to the bottom
E From the bottom to the top
An examination of the patient revealed abnormalities of the liver. Which embryonic germ layers have been damaged?
A * Endoderm middle sections of primary colon
B Endoderm posterior wall
C Foregut endoderm
D Mesonephritic duct
E Hindgut endoderm
As a result of stab wounds in the liver was damaged the hepatic artery, but the liver lobule blood continued to flow. Which
vessel provided blood flow in lobules?
A * Interlobular vein
B Interlobular artery
C Portal vein
D Sublobular vein
E Hepatic vein
In histological slide of parenchyma of the organ slices is presented, the shape is hexagonal prisms and consist of anastomosing
plates between which are sinusoidal capillaries that converge radially to the central vein. What is the anatomical organ has
given morphological structure?
A * Liver
B Pancreas
C Thymus
D Spleen
E Lymph node
During carbohydrate over-feeding of the animals in the cells of the liver we can find histologically a large amount of glycogen
granules. What group of cell structures includes glycogen?
A * Trophic inclusion
B Secretory inclusion
C Excretory inclusion
D Pigment inclusion
E Organelles with special functions
As a result of hepatotrophic poisons in hepatocytes was destroyed granular EPR. Synthesis of which components will be
change in the epithelium of the liver?
A * Albumin and fibrinogen
B Phospholipids
C Glycogen
D Cholesterol
E Vitamins
Digestive system. Pancreas.
A 50 years old patient complains with increased appetite, thirst, decreased body weight, weakness and fatigue. During
laboratory examination revealed increase the amount of sugar in the blood. Which cells dysfunctions are associated with the
development of this disease?
A * B cells
B A -cell
C Thyrocytes
D Pankreatocytes
E Lipotropocytes
In histological slide were studied exocrine portion of the pancreas. In the cells of the exocrine parenchyma contained secretory
granules with enzymes. How do they come (these enzymes) to the digestive tract?
A * Through duct system
B Through the bloodstream
C Fall into the lymph
D Aхonic transport
E Dendritic transport
During the study of pancreatic cells plasmolemma after exposure to drugs offenses were discovered in the structure of the
glycocalyx. What are the chemical components of the glycocalyx is composed of cells of the pancreas?
A * Oligosaccharides
B Proteins
C Lipids
D Mineral salts
E Water
In the histological slide of the pancreas we can find a group of cells. Some of them are centrally located and has basophilic
secretory granules. Their secret regulates carbohydrate metabolism. What kind of cells they are?
A * B cells
B PP cells
C A cells
D Adipocytes
E D cells
Respiratory System
Premature infants have syndrom of respiratory failure. Failure of what aerohematic barriere component underlies this
pathology?
A *Surfactant
B Capillary endothelium
C Basal membrane of endothelium
D Basal membrane of alveolocytes
E Alveolocytes
A patient with an acute rhinitis has hyperemia and excessive mucus formation in nasal cavity. What epithelial cells of mucous
membrane have the intensified activity?
A *Goblet cells
B Ciliated cells
C Microvillous cells
D Basal cells
E Endocrine cells
A patient was admitted to the hospital with an asphyxia attack provoked by a spasm of smooth muscles of the respiratory
tracts. This attack was mainly caused by alterations in the following parts of the airways:
A *Small bronchi
B Median bronchi
C Large bronchi
D Terminal bronchioles
E Respiratory part
During dissection (autopsy) of the 65 years old man, who suffered from lung disease pathological process was mainly localized
in the bronchi, where during the histological examination were clearly visible glands, cartilage islands and pseudostratified
epithelium. Which bronchi were changes?
A * Secondary bronchi
B Primary bronchi
C Tertiary bronchi
D Bronchioles
E Terminal bronchioles
In electronic microphotograph were detected cells of the alveoli, which are part of the blood-air-barrier (aero-hematic barrier).
What are these cells?
A * Respiratory epithelial cells of the alveoli
B secretory epithelial cells of the alveoli
C Alveolar macrophages
D Clara cells
E Ciliated epithelial
In 66 years old man was diagnosed a malignant epithelial tumor, originating from the medium-sized (secondary) bronchi. What
epithelium is the source of this tumor?
A *Pseudostratifed
B Stratified squamous nonkeratinized
C Stratified squamous keratinized
D Simple columnar
E Transitional
In the preterm baby were found sticking walls of the alveoli due to lack of surfactant. Please indicate dysfunction of which
cells of the wall of alveoli causes this condition.
A * Alveolocytes type II
B Fibroblasts
C Alveolocytes type I
D Alveolar macrophages
E Secretory Clara cells
In the alveoli of the lungs are present special cells through which gas exchange, they are part of the aero-hematic barrier. What
are these cells?
A * Alveolocytes I type
B Clara cells
C Alveolar macrophages
D Alveolocytes II type
E Epitheliocytes with microvilli
As a result of trauma 30 years old man’s nose damaged mucous membrane that covers the upper part of the nasal cavity. What
will be the result, what it led to?
A * Smperception odorous substances
B Violation of humidification
C Violation of secretory activity of goblet cells
D Violation of warm air
E Violation of warming and humidification
The patient has an inflammation process in the trachea, which captures the epithelium and lamina propria of the mucosa. What
epithelium has undergone a change?
A * Single layer ciliated
B Stratified squamous nonkeratinized
C Stratified cuboidal
D Transitional
E Simple cuboidal
In the biopsy of the mucosa of patients with bronchial asthma revealed a significant number of cells with multiple
metachromatic granules. Name this cell?
A * Tissue basophils
B Macrophages
C plasma cells
D reticulocytes
E fibroblasts
Study of a tubular organ revealed that its median membrane consists of solid hyaline rings. What epithelium lines mucous
membrane of this organ?
A *Multinuclear prismatic ciliated
B Monostratal prismatic glandular
C Monostratal prismatic with a border
D Multistratal squamous nonkeratinizing
E Monostratal cubical
In the chemical industry workers after inhalation of corrosive vapors death occurred in ciliated epithelial cells of the bronchi.
How the cells of the epithelium regeneration take place?
A * Basal cell
B Goblet cells
C Endocrine cells
D Ciliated cells
E cilia-free cells (Clara cells)
In the airway epithelium are present dome-shaped cilia-free cells, which are placed on the surface of microvilli. There
cytoplasm contains glycogen granules, lateral and apical Golgi complexes, elongated mitochondria, and a few secretory
granules. What is the name of these cells?
A * Clara cells
B Goblet cells
C Endocrine cells
D Cells without border
E Cambial cells
In histological slide of the trachea are muco-ciliary comprising low cells have oval or triangular shape. Their tops they reach
epithelial apical surface, in some cells occurs mitosis. What function do these cells?
A * Is a source of regeneration.
B Part of the muco-ciliary complex.
C Secretes mucus.
D Secretes surfactant.
E Producing biologically active substances.
After a prolonged inflammation of the mucous membrane of the nasal cavity of a patient observed changes in the epithelium.
What epithelium has undergone a change?
A * Simple pseudostratified
B Simple squamous
C Stratified squamous
D Stratified cuboidal
E Stratified columnar
Urinary system
A female patient with pyelonephritis was admitted to the urological department. Examination revealed an associated infection
accompanied by pyelovenous reflux. This complication was induced by affection of the following structure:
A *Fornical renal apparatus
B Excretory renal tracts
C Straight tubules
D Renal tubules
E Renal corpuscle
In course of an experiment the blood pressure of an animal had a stable rise by means of renal artery constriction.
Hyperfunctioning of what renal cells cause this effect?
A *Juxtaglomerular cells
B Podocytes
C Endotheliocytes
D Interstitial cells
E Thick spot cells
Electron micrograph of a kidney fragment presents an afferent arteriole with big cells under endothelium. These cells contain
secretory granules. Name this type of cells:
A *Juxtaglomerular
B Mesangial
C Smooth muscular
D Juxtavascular
E Interstitial
A histological specimen of a kidney shows a part of the distal tubule going between the afferent and efferent arteriole. The
cells building the tubule wall have dense nuclei; basal membrane is absent. Such structural formation is called:
A *Macula densa
B Juxtaglomerular cells
C Mesangial cells
D Juxtavascular cells
E A microphotography represents a fragment of cortical substance of a kidney. This fragment contains thick spot cells and
juxtaglomerular cells with big secretory granules. What kidney structure is represented?
A *Juxtaglomerular apparatus
B Renal corpuscle
C Filtering barrier
D Prostaglandin apparatus
E Choroid glomus
Electron-microscope investigation of cortical substance of a kidney reveals some structures lined with prismatic epithelium
that normally has brush border and deep plicae of plasmolemma in its basal part. There is a big number of mitochondrions
between these plicae. These structures belong to the following part of a nephron:
A *Proximal tubule
B Distal convoluted tubule
C Henle's loop
D Renal corpuscle
E Distal straight tubule
A patient ill with chronic glomerulonephritis has a disturbed incretory function of kidneys. It will result in the deficit of the
following blood corpuscles:
A *Erythrocytes
B Leukocytes
C Thrombocytes
D Leukocytes and thrombocytes
E Erythrocytes and leukocytes
In patients with chronic pyelonephritis there is no acidification of urine, so it is no bactericidal. Which structures of the kidney
injury occurred?
A * Dark cells of the collecting tubules
B In the glomeruli
C In podocytes capsules
D In the proximal convoluted tubules
E In the distal tubules
Malformations of the urinary system occur, according to statistics, 10-14% of infants. What are the sources of the development
of the urinary system?
A * Nonsegmental caudal mesoderm
B Dorsal mesoderm
C Visceral leaf of ventral mesoderm
D Parietal leaf of ventral mesoderm
E Germ mesenchyme
The patient was suffered from edema (swelling). A large amount of protein present in urine. About dysfunction of which part
of nephron does it mean?
A * Renal corpuscle
B Proximal convoluted tubule
C Distal convoluted tubule
D Downturn of the loop of Henle
E Ascending part of the loop of Henle
Urine was taken from 17 year old young man from a bladder using the catheter. Which epithelial cells of the urinary bladder
can be detected by microscopy?
A * Transitional
B Stratified nonkeratinized
C Simple columnar
D Simple cuboidal
E Stratified keratinized
Microscopic study of kidney biopsies found in its cortex tubules about 60 microns in diameter; their wall is formed by a high
cuboidal epithelium with abundant long microvilli. What structure is it?
A * Proximal tubules
B Distal tubules
C Capsule kidney cells
D Team rolls
E Loop of Henle
In histological slide of renal cortex can be seen renal corpuscles and tubules of nephrons. It is known that in the tubules of the
nephron reabsorption occurs. Which tissue of the nephron involved in this process ?
A * Epithelial
B Connective proper
C Reticular
D Mucous
E Cartilaginous
An important part of the renal filtration barrier is a three-layer basement membrane, which has a special network structure of
the middle layer. Where is this basement membrane?
A * In the renal corpuscles
B In capillaries of the perytubular capillary network
C In the proximal tubules
D In the small tubules
E In the distal tubules live
During the clinical examination in 35 -year-old women with kidney disease identified in urine blood cells, fibrinogen, which is
likely due to the renal filter. Which structures are composed this filter?
A * Endothelium of the capillaries of the glomerulus, three-layer basement membrane, podocytes
B Three basement membrane
C Endothelium of the capillaries, the basement membrane
D Podocytes, the basement membrane
E Endothelium, podocytes
In electron microscopy revealed the kidney tubules, which are lined by cuboidal epithelium. In the epithelium distinguish light
and dark cells. Light cells had organelles. These cells provide the reabsorption of water from primary urine in the blood. Dark
cells in structure and function resemble parietal cells of the stomach. What tubules represented in microscopic slide?
A *Collecting ducts
B Proximal convoluted tubules
C Distal tubules
D Ascending part of the loop of Henle
E Descending part of the loop of Henle
Male reproductive system
One of the phases of spermatogenesis observed changes in the nucleus and cytoplasm of spermatids, leading to the formation
of mature gametes. What phase of gametogenesis is it?
A * Formation.
B Maturation.
C Growth
D Reproduction
E Proliferation
In testes destructive changes observed in cells of seminiferous epithelium, which are already in the stage of formation spermatids and spermatozoa. By which cells can restore spermatogenesis?
A * Spermatogonia
B Primary spermatocytes
C Secondary spermatocytes
D Supporting cells
E Leydig cells
In histological slide of the testes in layers of connective tissue between the seminiferous tubules occur as vascular nests of pale
acidophilic cells with well-developed smooth endoplasmic reticulum, Golgi complex and mitochondria. What kind of cells
they are?
A * Interstitial cells
B Supporting cells
C Myoid cells
D Fibroblasts
E Spermatogonia
During puberty, the cells of male gonads begin producing the male sex hormone testosterone, which causes the appearance of
secondary sex characteristics. Which cells of male gonads produce this hormone?
A * Leydig cells
B Sustentocytes
C Sertoli cells
D Supporting cells
E Spermatozoa
Female reproductive system.
A patient underwent Caesarean section. During the operation a long incision was made in the uterus wall and the fetus was
extracted from uterus. Healing of the sutured myometrium will proceed in the following way:
A *Formation of a fibrous cicatrix
B Formation of smooth muscular tissue
C Formation of cross-striated muscle fibers
D Proliferation of myosatellitocytes
E Hypertrophy of smooth myocytes
A 35 y.o. patient diagnosed with sterility came to gynaecological department for diagnostic biopsy of endometrium.
Microscopic examination revealed that mucous membrane is edematous, uterine glands are convoluted and filled with thick
secreta. Such changes in the endometrium are caused by excess of the following hormon:
A *Progesterone
B Estrogen
C Testosterone
D Somatotropin
E ACTH
In the histological slide of the ovary, stained with hematoxylin-eosin, were identified a follicle in which follicular epithelial
cells located in 1-2 layers and have a cuboidal shape, around the oocyte membrane is visible bright red color. What is the
name of this follicle?
A * Primary
B Primordial
C Secondary
D Mature (Graafian)
E Atretic
Normal human embryo implantation can occur only with the corresponding changes of endometrium. Which cells of
endometrium will increase during this process?
A * Decidual cells
B Macrophages
C Endothelial cells
D Fibroblasts
E Neurons
In the cortex of the ovary histological examination determined mature tertiary follicle. In which period of oogenesis they were
formed?
A * Stage of great growth
B Stage of small growth.
C Maturation.
D Reproduction.
E Formation.
For early diagnosis of pregnancy in women using their urine. The presence of which hormones would likely indicate a
pregnancy?
A * Chorionic gonadotropin
B Estriol.
C Aldosterone.
D Testosterone.
E Progesterone.
As a result of inflammation in the fallopian tubes the moving of zygote was difficult. What type of epithelium was changed?
A * Simple columnar ciliated
B Stratified squamous nonkeratinized
C Pseudostratified
D Simple cuboidal
E Stratified squamous keratinized
496. Microscopic examination of the ovary, which was removed during operation were detected primordial and primary
follicles and corpus luteum in the stage of prosperity. In what kind of stage of ovarian-menstrual cycle are the female
reproductive organs with these structural features of the ovary?
A *Premenstrual
B Menstrual
C Preovulatory
D Postmenstrual
E Ovulatory
The 35 years old patient, diagnosed with infertility in the gynecological department, diagnostic endometrial biopsy was done.
Microscopic examination revealed that the mucosa with edema, uterine glands are meandering, filled with thick secret. What
hormone causes the following changes in the endometrium?
A * Progesterone
B Estrogen
C Testosterone
D Somatotropin
E ACTH
Stop bleeding after childbirth is associated with the action of hormones on the structures of the uterus. Which component of the
uterine wall takes part in this most of all?
A * Middle layer of myometrium
B Endometrium
C The inner layer of the myometrium
D The surface layer of the myometrium
E Perimetrium
Histology of the endometrium is characterized by the following features: thickening, edema, presence of tortuous glands with
enlarged lumen that secrete large amounts of mucus, mitosis in cells not observed, in stroma are available decidual cells. What
is the stage of the menstrual cycle corresponds to the described picture?
A * Secretory (premenstrual or luteal).
B Menstrual
C Regenerative
D Proliferative
E Relative calm
Microscopic examination of the female internal reproductive organs, which were removed during operation was found an
embryo constructed of two blastomeres. Name the place of its location in case of normal development.
A *Fallopian tubes, close to the ampulla
B Fallopian tubes, close to the uterus
C The cavity of the uterus
D Abdomen
E Ovary
By producing a number of hormones placenta plays a part of temporary endocrine gland. What hormone may be detected in
woman's blood on the 3rd or the 4th day after begin of implantation, that is used in medicine for early pregnancy detection?
A *Chorionic gonadotropin
B Somatostatin
C Progesterone
D Vasopressin
E Oxytocin
At 40 years old woman weak labor associated with weak contractility of the myometrium. What do you need to inject for the
help?
A *Oxytocin
B Hydrocortisone
C Dexamethasone
D Aldosterone
E Prednisone
In ovarian-menstrual cycle changes occur endometrial glands.
glands?
A *Simple unbranched tubular
B Simple branched tubular
C Simple unbranched alveolar
D Alveolar unbranched
E Alveolar-tubular branching
Which of the following types are appointed endometrium
In histological slide of the ovary presented rounded structure, glandular cells of which contain lipid droplets. Identify this
structure.
A * Yellow body
B Primordial follicles
C Primary follicle
D Mature follicle
E Atretic body
Stop bleeding after childbirth is due to the action of oxytocin on the uterus wall. Which layer of the uterus reacts to the action
of this drug?
A * Myometrium
B Endometrium
C Perimetrium
D Parametrium
E Submucosa